SOAL Masuk 2018 [PDF]

  • 0 0 0
  • Suka dengan makalah ini dan mengunduhnya? Anda bisa menerbitkan file PDF Anda sendiri secara online secara gratis dalam beberapa menit saja! Sign Up
File loading please wait...
Citation preview

SOAL Masuk 2018 1. Polypeptida hormone which helps gonadotrophin actions in theca and granulosa cells is : A. Insuline-like growth factors B. Anti-mullerian hormone C. Ephinephrine D. Glucagon E. Peptin 2. FSH function in follicular phase is : A. Avoid oocyte maturation inhibitor effect on oocyte B. Avoid domination of developing follicle C. Avoid premature lutenization of oocyte D. Avoid multiple oocyte maturation E. Avoid apoptosis of follicle 3. Estrogen produced by dominant follicle has an effect : A. Stimulates negative feedback on LH B. Inhibits positive feedback on LH C. Inhibits testerone aromatization on granulosa cell D. Stimulate FSH receptor formation in dominant follicle E. Inhibit LH receptor formation in dominant follicle 4. The basic philosophy of pulsatile gonadotrophin secretion is : A. To avoid downregulation B. To avoid positive feedback effect C. To avoid negative feedback effect D. To avoid accumulation hormone in target tissue E. To avoid degradation of hormone in bloodstream 5. Estrogen to estradiol conversion is facilitated by : A. 17beta hydroxysteroid dehidrogenase B. 3beta hydroxysteroid dehidrogenase C. P450 side chain cleavage D. P450 aromatase E. 5 alfa reductase 6. LH surge incite following events : A. Meiosis, theca cell luteinization, theca cell expansion, OMI synthesis B. Meiosis, theca cell luteinization, cumulus expansion, OMI inhibition C. Meiosis, granulosa cell luteinization, cumulus expansion, prostaglandin synthesis D. Meiosis, granulosa cell luteinization, theca cell expansion, luteinization inhibitor synthesis E. Meiosis, granulosa cell luteinization, granulosa cell expansion, prostaglandin inhibitor



7. The rise of FSH in preovulatory period has consequence : A. Release oocyte from its attachment in follicle, convert plasminogen to plasmin, LH receptor formation B. Bind oocyte to its attachment in follicle, convert plasminogen to plasmin, FSH receptor formation C. Bind oocyte to its attachment in follicle, convert plasminogen to prostaglandin, FSH receptor formation D. Release oocyte from its attachment in follicle, convert plasminogen to prostaglandin, LH receptor formation E. Release oocyte from its attachment in follicle, convert plasminogen to plasmin, prostaglandinreceptor formation 8. Acromosome reaction pproduces : A. Release of enzymes such as hyaluronidase and acrosin B. Separation between head and tail of spermatozoa C. Expansion of cumullus cells D. Separation between midplace and tail of spermatozoa E. Binding of spermatozoa with zona pellucida 9. Zone reaction will produce ; A. Zona pellucida become impermeable after fertilization of one spermatozoa B. Zona pellucida become permeable to allow easy penetration of spermatozoa C. Zona pellucida axpand to allow easy penetration of spermatozoa D. Zona pellucida become thin to allow easy penetration of spermatozoa E. Zona pellucida become thin to produce dense layer of cumullus cells 10. Capacitation is characteristized by : A. Acrosomal reaction, binding capabilities with zona pellusida, hipermotility B. Mitochondrial reaction, binding capabilities with cumullus, hipomotility C. Mitochondrial reaction, binding capabilities with cumullus with zona pellucida, hipomortility D. Acrosomal reaction, binding capabilities with cumullus, hipermotility E. Acrosomal reaction, binding capabilities with zona pellusida, hipomotility 11. The role of progesterone in pregnancy is : A. Substrate for glucocorticoid and mineralocorticoid production in fetal adrenal gland B. Fetal cholesterol pool C. Substrate for estrogen production in placental compartment D. Substrate for androgen production in maternal compartment E. Maternal cholesterol pool 12. Rapid conjugation of sex steroid hormone with sulfate compound is : A. To convert to more active form of sex steroid hormone B. To protect the fetus from adverse effect of abudant sex steroid hormone C. To ensure the plasental sulfatase enzyme in active condition D. To change to longer action from of steroid hormone E. To protect the maternal compartment from sex steroid hormone deficiency



13. Cortisol from maternal compartment cannot influence the fetal development due to : A. Its conversion to progesterone B. Its conversion to aldosterone C. Its conversion to cortisone D. Its conversion to estron E. Its conversion to DHAS 14. The fetus is protected from masculinization due to high level of androgen during pregnancy by : A. Aromatization maternal compartment and conjugation with sulfate compound B. Aromatization in placental compartment and conjugation with sulfate compound C. Aromatization in placental compartment and conjugation with hydroxy compound D. Aromatization in placental compartment and conjugation with glucuronyl compound E. Aromatization in fetal ovary compartment and conjugation with hydroxy compound 15. The decrease of estrogen and progesterone level in late luteal phase lead to: A. Increase of inhibin a secretion B. Decrease of pulsatile frequency secretion of GNRH C. Increase of pulsatile frequency secretion of GNRH D. Sustained feedback inhibition to hypothalamus until menstruasi phase E. Sustained feedback inhibition to hypothalamus until follicular phase 16. Progesterone production in pregnancy is dependent from A. Presence of fetus B. Presence of placental C. Presence of fetal ovary D. Presence of maternal ovary E. Presence of fetal DHAS production 17. Low estriol level in pregnancy correlation with A. Fetal macrosomia B. P45)c17 deficiency C. Adrenal hypofunction D. Adrenal hyperfunction E. 5-OH-steroid dehydrogenase deficiency 18. Wich of the following is the best explaination for breast development in a patient with androgen insensitivity? A. Gonadal production of estrogen B. Adrenal production of estrogen C. Breast tissue sensitivity to progesterone D. Peripheral convertion of androgens E. Autonomous production of breast-spesific estrogen 19. Miss mawar, 18 year old nulliparous complain of not having started her menses, her breast development is Tunner stage IV. She had blind vaginal pouch an no cervix. Which of the following is most likely diagnosis? A. Mullerian agenesis



B. Androgen insensitivity C. Both of the above D. Neither of the above E. Gonadal dysgenesis 20. Miss Beauty, 20 years old with 46, XY karyotype is noted to be a sexuality infantile phenotypic female and diagnosed as having gonadal dysgenesis. Which the following is the most important treatment for this patient? A. Progesteron therapy to reduced osteoporosis B. Estrogen and androgen therapy to enchance height C. Progesterone therapy to prevent endometrial cancer D. Gonadectomy E. Estrogen therapy to initiate breast development 21. Cytologic hormonal evaluation are sometime expressed by mean of indexes. Which the following index is not match for evaluating hormonal cytology? A. Crowded cell index B. Eosinophyllic index C. Maturation index D. Karyopyknotic index E. Mitotic index 22. Immunohystochemistry method can detect the specific antigen expresed by certain cells. Ussualy we chose the best of the following immunohystochemyitry method A. Direct method B. Peroxydase anti peroxydase method C. Avidin biotin complex method D. Toluidin method E. Massion trichrome method 23. Laparascopic ovarian surgery A. Should be limited to women with PCOS who have and clomiphene intolerance or resistence B. Result in live in 60-80 % of women C. Has similiar multiple pregnancy rates and miscarriage rate to ovalation induction with FSH D. Is associated with reduced sensitivity to ovulation induction with FSH E. All of the above 24. The mecanism of action of induction of ovulation following laparascopic ovarian drilling is A. Reduce serum FSH levels B. No change in inhibin levels C. Reduction in LH and androgen levels D. Reduction in circulating insulin and 1GFBP-1 levels E. All of the above 25. Which of the following statement is true about adenomyosis A. Adenomyotic tissue contains higher steroid receptor levels than normal myometrium B. Adenomyotic often associated with endrometriosis



C. The majority of adenomyosis cases are associated with adenomatous hyperplasia of the endometrium D. Hereditary occurance of adenomyosis is relative frequence E. Oral contraceptive pills are affective in reducing the uterine volume in adenomyosis



26. All of the following are possible etiologies of primary amenorrhea, except : A. Vaginal agenesis B. Asherman’s syndrome C. Mayer-rockitansky-kuster-hauser syndrome D. Testicular feminization E. Hypogonadotropic hypogonadism 27. Biomolecular technique which allows one small portion of DNA to be replicated and analyzed is A. Southern biot analysis B. Hybridization C. Polymerase chain reaction D. Cloning DNA E. Northern biotting 28. Most of the MHC antigens expression by trophoblast are A. Erithrocyte antigenic systems B. HLA-G C. uNK cells D. Placenthal alkaline phosphatase E. Rhesus-O 29. Endometrial receptivity, except : A. Has an implantation window of 4 days B. Is worse in stimulated IVF cycles C. Requires sufficient amounts of chorionic gonadotrophine D. Depends on matching blastocyst E. Involves up regulation of integrins 30. A 32-years old nulliparous woman has had oligomenorrhea since menarche. During the past 5 years she has experienced slow but progressive increase in hair on her face, back and forearms in the intermammary space, and on the back of her hands. Her voice has slowly deepened and temporal balding and clitoromegaly have developed. Which of the following is the most likely diagnosis? A. Polycystic ovary syndrome B. Cushing’s syndrome C. Stromal hyperthecosis D. Ovarian tumor E. Testis tumor 31. Fertility is reduced with increasing age of women because of a decreasing number of oocyte biology



this is/are true. Except : A. There is loss oocyte before birth B. The total number of oocytes both declines bi-exponentially with age and the loss accelerates around the age of 37 years C. Genetic factors are more important than environmental factors in determining the decline in the number of oocytes D. Recent evidence suggest thet ovarian stem cells are present in humans and are able to replenish in the number of oocytes E. Alterations in the neuroendocrine control with advancing age suggest a central contribution of the decline in the oocyte number 32. The known causes of reccurent miscarriage may be grouped as follows, expect : A. Balanced translocation B. Uterine septum C. Toxoplasma infection D. Bacterial vaginosis infection E. Thrombophilia related 33. Polypeptide hormone which helps gonadotrophin actions in theca and granullosa cells is A. Insulin-like growth factors B. Anti mullerian hormone C. Epinephrine D. Glucagon E. Leprin 34. In the management of adenomyosis, the following statements are true, except : A. The LNG-IUS has been shown both shrink adenomyotic deposits and overall improve menstrual symptoms B. Myometrial resection of adenomamyomata has been shown to improve menstrual symptoms in the majority of case series C. In the presence of rectovaginal adenomyamata, extensive surgery including hysterectomy is ussually required D. Succesful pregnancies have been reported after myometrial resection of adenomyomata E. During vaginal hysterectomy there appears to be a higher incidence of bladder injury in the presence of adenomyosis 35. The following are true regarding the induction of ovulation for woman with PCOS : A. Laparascopic ovarian drilling is as effective as low dose gonadotrophin therapy B. Laparascopic ovarian drilling is only effective for approximately six month following the operation C. Co-treatment with GNRH agonist and low dose gonadotrophins reduces the risk of OHSS



D. A pregnancy rate of approximately 80% is achieved with clomiphene citrate within 12 cycles E. The advantage of clomiphene over aromatase inhibitors is their lack of antioestrogen action on cervical mucous and endometrium 36. Which the following can be the etiology of either primary or secondary amenorrhea? A. Asherman’s syndrome B. Gonadal agenesis C. Sheehan’s syndrome D. Kallman’s syndrome E. Anorexia nervosa 37. Which of te following statement regarding hypothalamic amenorrhea is true? A. Patients with hypothalamic amenorrhea are hypogonadotropic-normogonadsm B. Patients with hypothalamic amenorrhea are hypogonadotropic-hypogonadsm C. Patients have a deficiency in SHBG secretion D. Nearly always be anorexia and weight loss E. Patients can be treated by clomiphene citrate



38. Which regard to endometriosis and infertility. The following statements are true, except: A. It is generally accepted that the laparascopic surgical treatment of minimal or mild endometriosis increases fecundity rate B. For moderate or severe endometriosis, results after laparascopy are comparable to those after laparatomy C. The endometrioma in most cases is a pseduocyst D. The consesnsus is that laparascopic surgical treatment of mild endometriosis restores fertility to normal E. There is ample evidence to suggest that ovarian follicle are present at the base of the endometriotic pseudocyst 39. The following condition is not a diagnostic criteria of PCOS according to Rotterdam criteria : A. Polycystic ovary appeareance on sonographic examination B. Exclusion of another type of hyperandrogenism C. Oligoovulation D. Hirsutism E. Obesity 40. Anovulation which occur after ovaritoxic agent chemotherapy was classified as : A. WHO anovulation class 1 B. WHO anovulation class 2 C. WHO anovulation class 3 D. WHO anovulation class 4 E. WHO anovulation class 1 and 3 41. The increase of insulin level in PCOS may lead to hyperandrogenism due to : A. Decrease of SHBG



B. Increase of SHBG C. Decrease of androgen D. Decrease of SHBG elimination E. Decrease of free androgen 42. Synergistically, insulin wil cooperate with following hormone to increase androgen production in PCOS : A. LH B. FSH C. Inhibin B D. Inhibin A E. Estradiol 43. In weight loss management in PCOS, the most important aspect is : A. Overall caloric restriction B. Carbohydrate restriction C. Fat rsetriction D. Activity restriction E. Appetite restriction



44. The future medical condition which may occur due to PCOS : A. Cervical malignancy B. Endometrial malignancy C. Osteoporosis D. Alzheimer disease E. Parkinson disease 45. If a PCOS woman still not ovulating after life style modification, weight loss, and clomiphen citrate administration, the next step will be : A. Metformin administration B. Progestin administration C. Gonadotrophin administration D. Aromatase inhibitor administration E. Combined oral contraception administration 46. In PCOS, the consequence serine phosphorylation disturbance is : A. Increase of Akt pathway activation B. Disturbance of Akt pathway C. PI3K activity is increased D. Disturbance of insulin binding to its receptor E. Decreased testosterone production 47. The evidence of genetic factor in PCOS is: A. 50% increase of PCOS incidence in siblings B. Patients of PCOS patient did not reveal any insulin resistence conditions C. PCOS patients have lower average birthweight D. PCOS patients have a hystory of maternal consumption of particular drugs E. PCOS patients have hystory of abnormal nurturing during their childhood period 48. The gold standart of insulin resistence measurement is :



A. QUICK1 method B. HOMA-IR method C. Anthropometric method D. Euglycemic clamp method E. Oral glucosa tolerance test method 49. Metformin therapy for PCOS correlated with : A. Increase of body mass index B. Increase of insulin sensitivity C. Increase of blood pressure D. Increase of LDL level E. Increase of fasting blood glucosa level 50. DHEA mostly produced by : A. Adipose tissue B. Adrenal tissue C. Ovary D. Hypophysis E. Liver 51. The important enzyme for androgen potency is : A. 17hidroxysteroid dehydrogenase B. Estradiol 2-hydroxylase C. 5 alpha reductase D. Tyrosin kinase E. Aromataase 52. Even after menopause most woman have circulating estrogen. It mainly originates from the aromatization of : A. Androstenedione to estrone by ovarian granulosa cells B. Androstenedione to estrone by ovarian theca cells C. Androstenedione to estrone by adipose tissue D. Testosterone to estradiol by adipose tissue E. Estradiol to estrone by adipose tissue 53. Estrogens are produced by the mother, fetus, and placenta. Which one of the following is true? A. Estradiol accounts for 80% of the estrogen produced during pregnancy B. Estriol is produced primarily by the placenta C. Anencephaly is associated with a normal level of estriol D. Estrogen supresses oxytocin secretion E. Estrone accounts for 80% of the estrogen produced during pregnancy 54. The decrease of progesterone in the end of menstrual cycle will lead to : A. The decrease of plasminogen activator inhibitor-1 activity B. The activation of metalloproteinases C. The blocking of metalloproteinase D. The decrease of prostaglandin activity E. The decrease of inflammation activity 55. The example of estrogen withdrawal bleeding is : A. Corpus luteum regression B. Withdrawal of progestin pills



C. The excessive effect of progestin D. Post-oophorectomy in follicular phase E. Estrogen dominance in chronic anovulation 56. The laparascopic ovarian drilling is the choice of ovulation induction method PCOS patient if : A. The basal LH level more than 10 mIU/ml B. Very low basal estradiol level C. Antiestrogen administration failed D. No ovulation after metformin administration E. Polycystic ovary morphology was seen with ultrasonography 57. Hyperprolactinemia may caused anovulation throught : A. Increase of continuous GnRH secretion B. Increase of inhibin activity in target organ C. Block the estrogen effect in target organ D. Block the FSH effect in target organ E. Block the GnRH pulsatile secretion 58. The abnormal uterine bleeding due to coagulophaty may only on seen as : A. Spotting B. Gum bleeding C. Postpartum bleeding D. Excessive bleeding since menarche E. Excessive bleeding during progestin contraception administration 59. In abnormal uterine bleeding, biopsy was not cosidered necessary if the endometrial thickness was: A. 5-12 mm B. >12 mm C. >14 mm D. < 8 mm E. < 4 mm 60. In unopposed estrogen condition, progestin therapy would reduce the severity of abnormal uterine bleeding throught mechanism : A. Inducing aromatase activity B. Inducing 5-alpha reductase activity C. Inducing estrogen receptor formation in endometrium D. Inducing 17-beta hydroxysteroid dehydrogenase and sulphotransferase to convert estradiol to estrone E. Inducing 17-beta hydroxysteroid dehydrogenase and sulphotransferase to convert estrone to estroadiol 61. The benefit of NSAID in abnormal uterine bleeding is : A. No interference to ovulation B. Increase the prostacyclin level C. Work in hypophysis-ovary axis D. Can be applied in patient with coagulopathy E. May lead to regular menstrual cycle



62. The indication for GnRH agonist therapy in management of uterine fibroids is : A. May allow for anemia correction in patient with severe anemia B. Remove the fibroid mass due to estrogen deprivation C. Reduce the endometrial thickness in unopposed estrogen condition D. Reduce the amount of blood transfusion during operation E. Avoiding the endometrial atrophy 63. With regard to strategies for preventing OHSS, the following statements are true, except : A. The use of the GnRH agonist long down regulation protocol is associated with a higher incidence of OHSS B. GnRH antagonist protocols do not cause even more severe OHSS C. Serum oestradiol levels are closely and positively correlated to OHSS, and can be used in isolation to predict OHSS D. Leutinizing hormone can be used in place of hCG to reduce the risk of OHSS E. Measeres to reduce the risk of OHSS include “coasting”, withholding hCG, early hCG administration and freezing all embryos



64. The following is true about hyperinsulinemia in women with PCOS, except : A. Hyperinsulinemia is common in obese women but does not occur in lean women B. It has been attributed to abnormal serine phosphorylation of the insulin receptor C. Hyperinsulinemia is more commonly associated with infertility than any other endocrine/metabolic factor D. Hyperinsulinemia stimulates ovarian androgen production E. Hyperinsulinemia is associated with an increased risk of early pregnancy loss 65. The following immunological function(s) are/is suppressed in women with endometriosis : A. Cytokine synthesis B. Antibody synthesis C. NK function D. Phagocytosis E. Leukocyte chemotaxis 66. The following are considered a mandatory part of investigation for menorrhagia : A. Full blood count B. Serum ferritin C. Coagulation screen D. Thyroid function E. Endometrial biopsy 67. The indication for estrogen therapy in abnormal uterine bleeding is : A. Bleeding due to chronic anovulation B. Bleeding due to coagulation defect C. Intermenstrual bleeding during progestin therapy D. Bleeding due to endometrial polyp E. Bleeding due to local inflammation from the IUD



68. PIBF, a protein mediating the immuno-modulatory effects of progesterone during pregnancy is known to, except : A. Induce the pregnancy-protective shift from Th2 dominance to Th1 dominance B. Advance the synthesis of a asymmetric antibodies C. Increase during normal pregnancy from 7 th to 37th weeks D. Supress NK cell activity E. Be produced extensively after organ transplantation 69. The following are characteristics of embryos at the blastocyst stage, except : A. This stage is reached at 5 or 6 days after fertilisation of the egg B. They are developing under the control of the embryonic genome C. They generally have higher implantation rates than cleavage stage embryos following in vitro fertilisation D. After fertilisation in vivo they are typically located in the fallopian tube E. In vitro culture of embryos to this dtage preferably involves use of sequental culture media 70. Hyperprolactinemia can cause secondary amenorrhea by : A. Inhibiting activity by interacting with hypothalamic dopamine and opioidergic systems B. Directly inhibiting ovarian activity C. Causing galactorrhea D. Increasing gonadotropin production E. Increasing inhibin production 71. Appropriate therapy for hyperinsulinemic women with polycystic ovary syndrome includes : A. Bromocriptine B. Troglitazone C. Metformin D. Clomiphene citrate E. Oophorectomy 72. In the treatment of women with PCOS which of the Following is true : A. The use of the combined oral contraceptive pill for the amelioration of the androgenic symptoms of PCOS is of greatest benefit to lean woman B. Firm evidence exist that early pregnancy treatment with metformin reduces the risk of miscarriage in women with PCOS C. Metformin does not cross the placenta and is therefore considered safe in pregnancy D. Metformin is more beneficial in the treatment of hirsutism in women with PCOS than sprinolactone E. Ovarian drilling (diathermy) is as effective long-term as gonadotrophin therapy at inducing ovulation in clomiphene resistent anovulatory women with PCOS 73. A 16 year old female with primary amenorrhea is tanner stage 1 for both breast and public



development. Serum FSH and LH levels are both elevated. Which of the following studies is indicated at this time : A. Dihydroepiandrosterone sulfate (DHEAS) level B. Testosterone level C. Computed tomography of the abdomen D. Chromosomal analysis E. Prolactin level 74. The abnormal uterine bleeding due to unopposed estrogen could be found : A. PCOS B. Low BMI C. Endometriosis patient D. Low ovarian reserve patient E. Post-oophorectomy patient 75. The estrogen therapy may be of benefit in the conditions : A. Breakhtrough bleeding in COC users B. PCOS related bleeding C. Endometrial hyperplasia is related bleeding D. Endometrial polyp related bleeding E. Chronic anovulation related bleeding 76. The management of abnormal uterine bleeding may consist of : A. Administration of copper-IUD B. Endometrial biopsy C. Administration of dopamine agonist drug D. Administration of calcium E. Administration of alendronate 77. The feature may be correlated with PCOS : A. Low insulin resistance B. Low level of DHAS C. Low level of future risk of cardiovascular disease D. Low SHBG E. Low estradiol 78. The drug which may have bone-losing effect is : A. GnRH agonist B. Estrogen C. Calcium supplement D. Exogenous FSH E. Exogenous hCG 79. A-28 year old nulliparaous woman request evaluation for hirsutism. The hair on her face, chest and back is of the fine, lanugo type. She has central obesity, abdominal striae, peripheral muscle wasting and dorsal neck fat pads. Which of the following would be the best laboratory test for evaluation of this patient : A. Serum testosterone B. Overnight dexamethasone suppression test



C. Serum dehydroepiandrosterone sulfate D. Serum ACTH E. Serum estrogen 80. A patient present with amenorrhea and galactorrhoea. Her prolactin levels are elevated. She is not and never has been pregnant. In additional to evaluating her for a prolactinoma, one also needs to evaluate for other causes that would increase prolactin such as elevated : A. Corticotrophin-releasing hormone B. Dopamine C. Gamma-aminobutyric acid (GABA) D. Throid stimulating hormone E. Thyrotropin-releasing hormone (TRH) 81. A 32-year old nulliparous woman has had oligomenorrhea since menarche. During the past 5 years she has experienced slow but progressive increase in hair on her face, back and forearms in the inter mammary space, and on the back of her hands. Her voice has slowly deepened and temporal balding and clitoromegaly have developed. Which of the following is the most likely diagnosis : A. Polycystic ovary syndrome B. Cushing’s syndrome C. Stromal hyperthecosis D. Ovarian tumor E. Testis tumor 82. Which of the following statement regarding hypothalamic amenorrhea is true : A. Patient with hypothalamic amenorrhea are hypogonadoytropic-normogonadism B. Patient with hypothalamic amenorrhea are hypogonadoytropic-hypogonadism C. Patients have a deficiency in SHBG secretion D. Nearly always be anorexia and weight loss E. Patients can be treated by clomiphene citrate 83. Since the work up of an elevated prolactin level can be expansive, it is appropriate to draw prolactin levels when the lowest values are to be except : A. Decrease sshortly after sleep B. Increase after ingesting high-carbohydrate meals C. Increase during stress D. Decrease during surgery E. Decrease after excercise 84. Which of the following statements regarding GnRH-stimulated LH secretion is accurate : A. It is enhanced by gonadotrope exposure to progesterone B. It is enhanced by gonadotrope exposure to testosterone C. It is enhanced by gonadotrope exposure to continuous GnRH D. It is enhanced by gonadotrope exposure to estrogen E. It is associated with steady LH release 85. The progressive sequence (with steps omitted) in the metabolism of steroid hormone is : A. Cholesterol-estradiol-testosterone-pregnolone B. Cholesterol-pregnolone-cortisol-estradiol



C. Cholesterol-pregnolone-estrone-androstenedione D. Cholesterol-androstenedione-pregnolone-estradione E. Cholesterol-pregnolone-androstenedione-estrone 86. All drug for medical treatment of hirsutism should seek to alter at least one of the major aspect of androgen metabolism, except : A. Decrease production B. Inhibit androgen receptor C. Increase SHBG D. Decrease the metabolic clearance rate E. Inhibit peripheral enzyme production 87. The New progestin A. Demonstrate decreased androgenicity B. May increase insuline resistance C. Decrease SHBG D. Demonsrate increase mineralocorticoid activity E. Cause markedly less breakthrough bleeding than traditional progestins 88. The use of contraceptive : A. Protects againts endometrial cancer B. A. Protects againts malignant breast cancer C. A. Protects againts thromboembolic desease D. A. Protects againts cardiovascular desease E. A. Protects againts cerebrovascuar desease 89. The dominant follicle : A. Has a lesser content of FHS receptors B. Contains granulosa cells with greater rate of proliferation than its cohorts C. Containts low concentration of local autocrine a paracrine peptides D. Is selected ... E. Is always........ 90. Inhibin : A. Is produced by FSH stimulated granulosa cells B. Secretion is not inhibited by GnRH C. Exist as only one isoforms D. Rises throught the follicular phase to reach a midcycle peak followed by a greater mildluteal peak E. Converted to activin by theca cells 91. The pituitary secretion of FSH is influenced by the balance of : A. IGF secretion B. Aromatase secretion C. Progesterone secretion D. GnRH secretion E. Testosterone secretion 92. The role of androgen in folliculogenesis A. At high concentration enhance their own aromatization and contribute to estrogen production B. Increase the FSH receptor content of the follicle



C. At higher level cause follicular atresia D. May serve as substrate for progesterone E. always produced by adrenal gland 93. The preovulatory follicle : A. As it approaches maturity produces decreasing amounts of estrogen B. Contains granulosa cells which enlarge acquire lipid inclusions and theca which becomes vacuolated and vascular C. Does not produce progesterone prior to ovulation D. Containts an oocyte which resumes mitosis E. Laways 18 mm in diameter 94. Ovulation A. Is most reliably predicted by the onset of the LH surge which occurs 24-36 hours prior to response B. Occurs approximately 10-12 hours after the estradiol peak C. Requires a threshold of LH conventration to be maintained for prolonged period inorder for full maturation of the oocyte to occur D. Involves no variation in timing from cycle to cycle within the same women E. Always occur in early morning 95. The enzyme that take an important role in endometrial shedding A. Metalloproteinase B. Aromatase C. 5-alpha reductase D. 17-hidroxysteroid dehydrogenase E. Tyrosin kinase



96. The classification system used of abnormal uterine bleeding is : A. ASRM classication system B. ENZIAN system C. PALM COEIN system D. ESGE system E. Rotterdam system 97. Amenorrhea due to structures exercise is classified as A. Primary amenorrhea B. Drug .... Amenorrhea C. Ovarian failure amenorrhea D. Galactorrhea amenorrhea E. Hypothalamic amenorrhea 98. In anovulatory women, if the hormonal assay result is normal, then it classified as : A. Group I anovulation B. Group II anovulation C. Group III anovulation D. Group IV anovulation E. Combined Group II and IV anovulation 99. Clomiphene citrate only can be used in



A. Group I anovulation B. Group II anovulation C. Group III anovulation D. Group IV anovulation E. Combined Group II and IV anovulation 100. Amenorrhea which occur in woman who already had menarche is called : A. Primary amenorrhea B. Drug induce amenorrhea C. Ovarian failure amenorrhea D. Galactorrhheic amenorrhea E. Secondary amenorrhea



SOAL UNAS 1 23 MARET 2019



1. The complete set of DNA in an organism is called: a) A chromosome b) A gene c) A proteome d) A genome e) A nucleus



2. A contiguous region of DNA that can encode a protein product and contains within its regulatory sequences that regulate its expression is called: a) A genome b) A gene c) A chromosome d) A proteome e) A nucleus



3. A package of genetic material, consisting of a DNA molecule to which are attached large numbers of proteins that maintain chromosome structure and play a role in gene expression is calle a) b) c) d) e)



A chromosome A genome A gene A proteome A nucleus



4. The biologic effects of major steroid hormones are determined mainly by: a) Their affinity to albumin b) Their clearance rate c) Their production rate d) Their unbound portion e) Their capability to enter reservoir organs



5. The major androgen products of the ovary are: a) b) c) d) e)



Androstenedione and testosterone Dehydroepiandrosterone sulfate and androstenedione Testosterone and dihydrotestosterone Dehydroepiandrosterone sulfate and dihydrotestosterone Androstenedione and dihydrotestosterone



6.In the major sex-steroid receptors, the role of heat shock protein (HSP) is: a) To enhance the binding potential of the receptor, therefore prevent early separation with hormone b) To stabilize the clearance rate of the hormone-receptor complex and ensure the biological effects is optimally achieved c) To stabilize and protect the receptor and maintain the DNA binding region in inactive state d) To prevent the downregulation effect due to excessive exposure from the hormone e) To enhance the translation process in the DNA binding region.



7. The major factor in the potency differences among the various estrogens (estrone, estradiol, estriol) is: a) The length of time the estogen is bound with the SHBG b) The length of time the estrogen-SHBG complex cleared from the circulation c) The length of time of the estrogen-receptor complex occupies the ribosome to synthesize proteins d) The length of time of the estrogen-receptor complex occupies the nucleus e) The length of time of dissociation between estrogen and SHBG



8. The conformational changes of steroid receptors is induced by: a) Steroid hormone and lead to dissociation of HSP90 b) HSP90 and lead to the inactivation of steroid receptors c) Steroid hormone and lead to the formation of dimers without dissociating HSP90 d) TAF2 and may be achieved without full hormone binding e) TAF1 and may be achieved without full hormone binding



9. The main characteristics of non-genomic, extranuclear estrogen stimulation activity is: a) Gene expression and lead to specific protein synthesis b) Rapid response and associated with growth factors and G-protein messenger



c) Classic dimerization after ligand binding d) Gradual response involving receptor and ligand binding, then activation of intracellular organelles e) Activation of the estrogen receptors pathway through second messenger involving phosphorylation of estrogen receptor and coregulator proteins



10. The characteristics of gonadotrophin secretion in childhood period is: a) High level of gonadotrophin with maximal hypothalamic suppression b) Low level of sex steroid hormone with maximal hypothalamic suppresion c) Low level of gonadotrophin with no hypothalamic suppression d) Low level of gonadotrophin with maximal hypothalamic suppression e) High level of sex steroid hormone with no hypothalamic suppresion



11. The following reactions are occur as consequences of estrogen and progesterone withdrawal: a) The process of regeneration, vasomotor reactions, and tissue growth b) The process of necrosis, vasomotor reactions, and tissue loss c) The process of regeneration, tissue loss, and vascular constriction d) The process of apoptosis, vasomotor reactions, and tissue loss e) The process of apoptosis, tissue regeneration, and vascular breakdown



12. The following substances are induced after estrogen and progesterone withdrawal in menstrual cycle: a) IGF-I b) Aromatase c) DNA transcriptase d) Metalloproteinase e) Inhibin A



13. Prolactin secretion is mainly controlled by: a) Stimulatory control of GnRH b) Inhibitory control of hypothalamic dopamine c) Stimulatory control of TRH d) Inhibitory control of kisspeptin e) Inhibitory control of ACTH



14.The short feedback loop in neuroendocrine system means: a) A negative feedback of pituitary hormones on their secretions, presumably via inhibitory effects on releasing ho b) A feedback effects of circulating effects of target gland hormones c) A negative feedback from adjacent cells in the same tissue d) A negative feedback of pituitary hormones on their secretions, presumably via inhibitory effects on releasing hormones in the hypothalamus. e) A negative feedback from endocrine gland cells to their neighboring glands



15. Biomolecular technique which allows one small portion of DNA to be replicated and analyzed is: a) b) c) d) e)



Polymerase Chain Reaction Southern Blot Analysis Hybridization Cloning DNA Northern blotting



16. Polypeptide hormone which helps gonadotrophin actions in theca and granulosa cells is: a) b) c) d) e)



Insulin-like growth factors Anti-mullerian hormone Epinephrine Glucagon Leptin



17. The important enzym for androgen potency is: a) 17 hidroxysteroid dehydrogenase b) estradiol 2-hydroxylase c) tyrosin kinase d) 5 alpha reductase e) aromatase



18. The decrease of progesterone in the end of menstrual cycle will lead to: a) b) c) d) e)



the activation of metalloproteinases the decrease of plasminogen activator inhibitor-I activity the blocking of metalloproteinases the decrease of prostaglandin activity the decrease of inflammation activity



19. The major function of zona pellucida during fertilization is: a) A selective barrier for every sperm to protect from premature fertilization b) Contains ligand for sperm which is species-specific c) Contains antibody for sperm to protect multiple fertilization d) A nonselective chemoattractants for sperm e) Contains specific proteins to protect from oocyte postmaturity



20. The most optimal timing of endometrial receptivity is characterized by  a) b) c) d) e)



Sperm-induced cortical reactions Estrogen induced spiral artery remodellings Progesterone-induced pinopode formations Sperm-induced acrosomal reactions Inhibin-B induced FSH decrease



21. The function of pinopodes is: a) To provide the wide surface for blastocyst implantation



b) To enhance endometrial fluid production to promote embryo implantation c) To create more vascular area to promote more cytokine production d) To absorb the fluid and forcing blastocyst to be in contact with the endometrium e) To block the cortical reactions therefore prevents polyspermia reactions.



22. The critical features of transition of estrogen suppression to LH stimulation in the midfollicular phase are: a) The estradiol bind by SHBG and hepatic clearance b) The estradiol production rate and hepatic clearance rate c) The estradiol concentration and duration of sustained elevation d) The estradiol concentration and the rate of SHBG conjugation e) The estradiol production rate and its affinity with their receptors in granulosa cells



23. The characteristics of gonadotropin pulsation during follicular phase are: a) Less frequent and greater in amplitude b) More frequent but smaller in amplitude c) More frequent and greater in amplitude d) Less frequent and smaller in amplitude



e) More frequent throughout the cycle with logarithmic amplitude



24. The majority of follicular development time until reaching the ovulatory stage was spent on a) b) c) d) e)



Gonadotrophin dependent stage Cyclic recruitment stage Gonadotrophin independent stage Domination stage Resting pool stage



25. The continuing growth of antral follicles in response to FSH is known as: a) b) c) d) e)



Selection stage Domination stage Recruitment stage Final maturation stage Atresia stage



26. The main function of AMH in the follicular development is: a) To promote granulosa cell differentiation b) To inhibit primordial follicel growth c) To stimulate the follcle responsiveness to FSH d) To stimulate the follicle responsiveness to LH e) To induce final oocyte maturation



27. The FSH rise in few days before menses is triggered by:



a) teep rise of midcycle steroidogenesis b) A decline in luteal phase steroidogenesis and inhibin A secretion c) The sustained high level of progesterone during luteal phase d) The decline of ovarian steroid after LH surge is achieved e) The activation of inhibin B in midfollicular phase



28. The rate of androgen conversion to estrogen in the granulosa cells is limited by: a) b) c) d)



The aromatase activity The long negative feedback of estrogen to hypothalamus The intracrine regulations by growth factors The presence of abundance cholesterol as basic substrate e) The maturity and differentiation of theca cells



29. In the presence of FSH, the dominant substance in the follicular fluid is: a) b) c) d) e)



Androgen Amino acids Estrogen Growth factors Cumullus cells



30. During the folliculogenesis, if plasma LH is prematurely elevated in the follicular phase, it may lead to: a) The decrease of granulosa cell mitotic activity b) The rise of intrafollicular estrogen



c) The rise of intrafollicular growth factors d) The decrease of intrafollicular androgen e) The rise of long positive feedback to estrogen activity



31. The selection of dominant follicle is the result of: a) The effect of estrogen on pituitary secretion of FSH b) The effect of estrogen on granulosa cells actions of growth factors c) The effect of production of FSH receptors in the follicle d) The effect of proliferation of LH receptors in the follicle e) The effect of estrogen on surrounding granulosa cells



32. The highest concentration of AMH was found in: a) Atretic follicles b) Small antral follicles c) Dominant follicles d) Early primordial follicles e) Preovulatory follicles



33. The serum AMH level can be accurately measured in any day of menstrual cycle because of: a) It is secreted steadily by all follicular cells b) It is stimulated by high level of ovarian steroids in the midcycle period c) It is stimulated by GnRH throughout the cycle d) It is not affected by gonadotrophins or sex steroids e) It is not affected by the rate of atresia in every cycles



34. During periovulatory period, LH surge will trigger plasminogen activators produced by granulosa cells will activate plasminogen to plasmin in follicular fluid to activate: a) b) c) d) e)



Progesterone synthesis Collagenase system Androgen synthesis Long feedback system Second meiosis



35. The mechanism that may limit the duration of LH surge is: a) b) c) d) e)



The decrease of midcycle FSH level The progressive rise of estrogen via negative feedback The decrease of inhibin-B secretion The progressive rise of progesterone via negative feedback The intrafollicular rise of insulin-like growth factor II



36. This drug may inhibit the ovulation process in the ovulatory period: a) b) c) d) e)



Dinoprost Folic acid Oxytocin Ketoprofen Cholecalciferol



37. The lifespan and steroidogenic capacity of the corpus luteum are dependent on: a) Continued tonic LH secretion b) Positive feedback of preovulatory estrogen rise



c) Continued tonic FSH secretion d) Negative feedback of luteal phase progesterone to hypothalamus e) Continued rise of inhibin-B secretion



38. During the luteal phase, the initiation of new follicular growth is inhibited by: a) The low level of of ovarian steroids due to demise of the corpus luteum b) The low level of gonadotrophin due to negative feedback of estrogen, progesterone, and inhibin-Ac. c) The declining level of LH immediately after its surge d) The declining level of estrogen before LH surge is reached e) The increasing level of FSH in the luteofollicular transition



39. The important aspect of corpus luteum rescue by hCG is to: a) b) c) d) e)



Suppress tissue inhibitors of metalloproteinases (TIMP) Increase the aromatase expression Prevent matrix metalloproteinase expression Increase plasmin expression Prevent steroidogenesis synthesis



40. The process of nuclear division in all somatic cells is called  a) b) c) d) e)



Meosis Mitosis Prophase Anaphase Telophase



41. Acrosome reaction produces: a) Separation between head and tail of spermatozoa



b) c) d) e)



Expansion of cumullus cells Separation between midpiece and tail of spermatozoa Release of enzymes such as hyaluronidase and acrosin Binding of spermatozoa with zona pellucida



42. Zone reaction will produce: a) Zona pellucida become impermeable after fertilization of one spermatozoa b) Zona pellucida become permeable to allow easy penetration of spermatozoa c) Zona pellucida expand to allow easy penetration of spermatozoa d) Zona pellucida become thin to allow easy penetration of spermatozoa e) Zona pellucida become thin to produce dense layer of cumullus cells



43. Capacitation is characterized by: a) Mitochondrial reaction, binding capabilities with cumullus, hypomotility b) Mitochondrial reaction, binding capabilities with zona pellucida, hypomotility c) Acrosomal reaction, binding capabilites with zona pellucida, hypermotility d) Acrosomal reaction, binding capabilites with cumullus, hypermotility e) Acrosomal reaction, binding capabilites with zona pellucida, hypomotility



44. The role of progesterone in pregnancy is:



a) Fetal cholesterol pool b) Substrate for glucocorticoid and mineralocorticoid production in fetal adrenal gland c) Substrate for estrogen production in placental compartment d) Substrate for androgen production in maternal compartment e) Maternal cholesterol pool



45. Rapid conjugation of sex steroid hormone with sulfate compound is: a) To protect the fetus from adverse effect of abundant sex steroid hormone b) To convert to more active form of sex steroid hormone c) To ensure the placental sulfatase enzyme in inactive condition d) To change to longer action form of steroid hormone e) To protect the maternal compartment from sex steroid hormone deficiency



46. Cortisol from maternal compartment cannot influence the fetal development due to: a) b) c) d) e)



Its conversion to progesterone Its conversion to aldosterone Its conversion to cortisone Its conversion to estrone Its conversion to DHAS



47. The fetus is protected from masculinization due to high level of androgen during pregnancy by:



a) Aromatization in maternal compartment and conjugation with sulfate compound b) Aromatization in placental compartment and conjugation with sulfate compound c) Aromatization in placental compartment and conjugation with hydroxy compound d) Aromatization in placental compartment and conjugation with glucuronyl compound e) Aromatization in fetal ovary compartment and conjugation with hydroxy compound



48. The decrease of estrogen and progesterone level in late luteal phase lead to: a) b) c) d)



Increase of inhibin A secretion Decrease of pulsatile frequency secretion of GnRH Increase of pulsatile frequency secretion of GnRH Sustained feedback inhibition to hypothalamus until menstrual phase e) Sustained feedback inhibition to hypothalamus until follicular phase



49. Polypeptide hormone which helps gonadotrophin actions in theca and granulosa cells is:49. Polypeptide hormone which helps gonadotrophin actions in theca and granulosa cells is:



a) b) c) d) e)



Anti-mullerian hormone Epinephrine Insulin-like growth factors Glucagon Leptin



50. The role of androgen in folliculogenesis: a) At high concentration enhance their own aromatization and contribute to estrogen production b) Increase the FSH receptor content of the follicle c) May serve as substrate for progesterone d) At higher level cause follicular atresia e) Always produced by adrenal gland



51. The preovulatory follicle a) As it approaches maturity produces decreasing amounts of estrogen b) Contains granulosa cells which enlarge and acquire lipid inclusions and theca which becomes vacuolated and vascular c) Does not produce progesterone prior to ovulation d) Contains an oocyte which resumes mitosis e) Always 18 mm in diameter



52. Ovulation a) Is most reliably predicted by the onset of the LH surge which occurs 24-36 hours prior to response b) Occurs approximately 10-12 hours after the estradiol peak c) Requires a threshold of LH concentration to be maintained for prolonged period in order for full maturation of the oocyte to occur d) Involves no variation in timing from cycle to cycle within the same woman e) Always occur in early morning



53. The enzyme that take an important role in endometrial shedding: a) b) c) d) e)



aromatase 5alpha reductase metalloproteinase 17 hidroxysteroid dehydrogenase tyrosin kinase



54. The estrogen during pregnancy influence: a) b) c) d) e)



Placental development Progesterone production Maternal blood pressure regulation Fetal brain development Fetal gender assignment



55. The substrate for fetal adrenal gland production of glucocorticoids and mineralocorticoids is: a) Androgen



b) c) d) e)



Progesterone Estrogen Somatomammotropin Prostaglandin



56. The circulating level of SHBG is inversely corelated with: a) b) c) d) e)



Body height Estrogen production Body weight Hypothalamic signaling Long feedback to hypothalamus



57. The main difference between TAF-1 and TAF-2 is: a) TAF-2 can stimulate transformation change without hormone binding b) TAF-1 is located in the hormone binding domain of estrogen receptor c) TAF-1 can stimulate transcription without hormone binding d) TAF-2 is located in the regulatory domain of estrogen receptor e) TAF-1 need strong hormone binding to induce transcription process



58. The “chaperone” protein that keeps the estrogen receptor in inactive form is: a) b) c) d) e)



HSP 90 TAF-1 TAF-2 SHBG IGF-II



59. Polypeptide hormone which helps gonadotrophin actions in theca and granulosa cells is: a) b) c) d) e)



Anti-mullerian hormone Epinephrine Glucagon Insulin-like growth factors Leptin



60. FSH function in follicular phase is: a) b) c) d) e)



Avoid oocyte maturation inhibitor effect on oocyte Avoid domination of developing follicle Avoid premature lutenization of oocyte Avoid apoptosis of follicle Avoid multiple oocyte maturation



61. Estrogen produced by dominant follicle has an effect: a) b) c) d) e)



Stimulates negative feedback on LH Inhibits positive feedback on LH Stimulate FSH receptor formation in dominant follicle Inhibits testosterone aromatization on granulosa cell Inhibit LH receptor formation in dominant follicle



62. The basic philosophy of pulsatile gonadotrophin secretion is: a) b) c) d) e)



To avoid downregulation To avoid positive feedback effect To avoid negative feedback effect To avoid accumulation of hormone in target tissue To avoid degradation of hormone in bloodstream



63. Estrone to estradiol conversion is facilitated by: a) b) c) d) e)



3 beta hydroxysteroid dehidrogenase 17 beta hydroxysteroid dehidrogenase P450 side chain cleavage P450 aromatase 5 alfa reductase



64. LH surge incites following events: a) Meiosis, theca cell luteinization, theca cell expansion, OMI synthesis b) Meiosis, theca cell luteinization, cumulus expansion, OMI inhibition c) Meiosis, granulosa cell luteinization, cumulus expansion, prostaglandin synthesis d) Meiosis, granulosa cell luteinization, theca cell expansion, luteinization inhibitor synthesis e) Meiosis, granulosa cell luteinization, granulosa cell expansion, prostaglandin inhibition



65. The rise of FSH level in periovulatory period has consequence: a) Bind oocyte to its attachment in follicle, convert plasminogen to plasmin, FSH receptor formation b) Bind oocyte to its attachment in follicle, convert plasminogen to prostaglandin, FSH receptor formation



c) Release oocyte from its attachment in follicle, convert plasminogen to prostaglandin, LH receptor formation d) Release oocyte from its attachment in follicle, convert plasminogen to plasmin, LH receptor formation e) Release oocyte from its attachment in follicle, convert plasminogen to plasmin, prostaglandin receptor formation



66. The loss of oocytes during atresia is done through: a) b) c) d) e)



Necrobiosis process Necrosis process Inflammation process Apoptosis process Entosis



67. The characteristics of secretory phase endometrium is: a) b) c) d) e)



Continue to grow after ovulation Progressive tortuosity of the endometrial glands Increase in mitosis Increase in DNA synthesis Progressive decrease of estrogen receptors



68. These events is caused by high progesterone level in the endometrium: a) b) c) d)



Prostaglandin and VEGF expression Perivascular growth and decidualization Cytokine and MMP expression Leukocyte infiltration and vasoconstriction



e) Activation of caspase and vasodilatation



69. These events is caused by progesterone withdrawal in the endometrium: a) b) c) d) e)



Suppression of prostaglandin and MMP expression Suppression of caspase activities Perivascular growth and gland proliferation Increase in MMP and tissue breakdown Reepithelization of endometrium



70. The menstrual flow stops as a result of: a) b) c) d) e)



Progesterone induced healing Prolonged vasoconstriction of radial arteries Activation of MMP systems Autodigestion of endometrial glands Activation of caspase enzymes



71. This condition is associated with the increase of neuropeptide Y level: a) b) c) d) e)



Undernutrition Obesity Lack of sleep Exposure of dark Intermitten fasting



72. The suppression of gonadotrophins by GnRH agonist can be used in the treatment of: a) Polycystic ovary syndrome



b) c) d) e)



Precocious puberty Premenopausal syndrome Premenstrual syndrome Recurrent pregnancy loss



73. The GnRH antagonists can be used in the treatment of: a) b) c) d) e)



Threatened abortion Premenopausal syndrome Premenstrual syndrome Precocious puberty Recurrent pregnancy loss



74. The characteristics of gonadotrophin secretion during prepubertal period is: a) Cyclic and showed different patterns between follicular and luteal phase b) Low but still demonstrate the irregular pulse c) Sustained high level throughout the cycle d) Sustained high level throughout the prepubertal period e) Low and correlated with decreasing concentration of DHEA



75. The central inhibitory signal that restraint GnRH pulsatile secretion is: a) b) c) d) e)



Prolactin Endorphin GABA Dynorphin Oxytocin



76. Progesterone production in pregnancy is independent from: a) b) c) d) e)



Presence of fetus Presence of placenta Presence of fetal ovary Presence of maternal ovary Presence of fetal DHAS production



77. Hormones with specific binding protein are: a) b) c) d) e)



Thyroid and insulin Thyroid and prolactin Thyroid and epinephrin Thyroid and testosteron Thyroid and gonadotrophin



78. Hyperprolactinemia may caused anovulation through: a) b) c) d) e)



increase of continuous GnRH secretion increase of inhibin activity in target organ block the estrogen effect in target organ block the FSH effect in target organ block the GnRH pulsatile secretion



79. Which of the following factors is the most influence for formation of the ext genitalia :  a) Genetic sex of the embryo b) A normal urogenital sinus



c) The sperm fertilizing the egg (carrying a Y chromosome) d) The presence of fetal androgens e) Maternal hormonal levels



80. The absence of the vagina is common in a) b) c) d) e)



Congenital adrenal hyperplasia in a female infant Turner syndrome Association with an absent or rudimentary uterus Drug-induced fetal masculinization of a female infant Gonadal dysgenesis



81. If germ cells fail to enter the developing genital ridge, which of the following may occur? a) b) c) d) e)



Ovarian teratomas Ectopic pregnancy Ovarian choriocarcinoma Gonadal agenesis Testicular feminization



 82. The paramesonephric ducts will form  a) b) c) d) e)



The prostatic utricle Seminal vesicles Oviducts, uterus, and upper vagina Upper vagina only The ureters



83. The correct order of cell layers surrounding an ovarian follicle from the oocyte outward is  a) b) c) d) e)



Zona pellucida, granulosa, theca interna Granulosa, theca interna, zona pellucida Theca interna, zona pellucida, granulosa Theca interna, granulosa, zona pellucida Zona pellucida, theca interna, granulosa



84. Most of the MHC antigens expression by trophoblast are a) b) c) d) e)



Erithrocyte antigenic systems HLA-G uNK cells Placenthal alkaline phosphatase Rhesus-D



85. Which of the following changes will maintain the corpus luteum ? a) b) c) d) e)



Decreased LH Increased FSH Increased hCG Decreased inhibin B Increased progesteron



86. This statement about GnRH secretion is correct, a) b) c) d)



The amplitude is highest at the early follicular phase. Have lowest frequencies at luteal phase. Have highest amplitude at late follicular phase. Less frequencies at pre ovulatory than early follicular phase.



e) Lowest amplitude at luteal phase.



87. This statement is correct about Prolactin, a) b) c) d) e)



Produced by lobus posterior pituitary. Produced only at the time of lactation. Suppress GnRH secretion. Stimulate LH production. The production stimulated by dopamine.



88. This statement is correct about gonadotropin production, a) b) c) d) e)



Is equal with serum gonadotropin level. Stimulate by inhibin-A Estrogen stimulate LH production. Stimulate by low level serum progesterone. Higher at mid luteal phase.



89. This statement is correct about serum gonadotropin level, a) Controlled by hypothalamus. b) Stimulate by low level serum progesterone at early follicular phase. c) Suppressed by high level serum estrogen, at mid cycle. d) Stimulate by activin. e) Suppressed by inhibin-B



90. The role of LH at late follicular phase is, a) Stimulate OMI production.



b) c) d) e)



Stimulate granulose cells progesterone production. Inhibit synthesis of progesterone. Inhibit synthesis of inhibin-A Suppress synthesis of androgen.



91. The requirement of serum estrogen level for getting LH surge , minimally  a) b) c) d) e)



100 pg/mL 200 pg/mL 300 pg/mL 400 pg/mL 500 pg/mL



92. The time of ovulation is: a) b) c) d) e)



36 hours from the beginning of LH surge 36 hours from the peak of LH surge 36 hours from the beginning of FSH sure 36 hours from the peak of FSH surge. 36. Hours from the peak of mid cycle estradiol level



93. The growth of public and axillary hair at puberty is due to an increased production of: a) Gonadal androgens



b) c) d) e)



Gonadotropins GnRH Estrogen Adrenal androgen



94. Hormone that act on the cell they are secreted from, are called : a) b) c) d) e)



Endocrine Paracrine Autocrine Neurocrine Exocrine



95. The primary genetic signal determining the direction of gonadal differentiation in mammals: a) b) c) d) e)



SRY TDF XY SF1 Akt-1



96. Vaginal epithelium and the fibromuscular wall of the vagina originate from which of the following respectively? a) b) c) d)



Mesonephric duct and endoderm of the urogenital sinus Mesonephric duct and the uterovaginal primordium Endoderm of the urogenital sinus and the mesonephric duct Endoderm of the urogenital sinus and the urovaginal primordium e) Endoderm of the urogenital sinus and the paramesonephric ducts



97. Which of the following is the principal site of production of sex hormone-binding globulin (SHBG)? a) b) c) d) e)



Kidney Pituitary Liver Hypothalamus Granulosa cell



98. Follicle stimulating hormone (FSH) : a) Is a steroid hormone b) Is produced by the chromophil cells of the posterior lobe of the pituitary gland c) Stimulates the interstitial cells of Leydig in the male d) Has alpha-subunits which are the product of the same gene that controls the alpha-subunits of luteinizing hormone (LH) and thyroid stimulating hormone (TSH) e) Serum levels are decreased during the climacteric



99. Luteinizing hormone (LH) : a) Is bound to plasma proteins b) Secretion is stimulated by rising levels of oestrogen c) Production is increased by sustained continuous administration of gonadotrophin releasing hormone analogues d) Inhibits testosterone production in the female e) Surge occurs within 36 hours after ovulation



100. During growth of the embryo, the inner cell mass is referred to as the embryonic disk. This embryonic disk differentiates initially into a) b) c) d) e)



1.



Decidua Cytotrophoblast Ectoderm and endoderm Syncytiotrophoblast Mesoderm



The mitosis is: A. The process of nuclear division in gamet cells B. The process of nuclear division in somatic cells C. The process of nuclear division in oocytes D. The process of nuclear division in spermatozoa E. All of above is true



2.



While circulating in the blood, 10–30% estradiol and testosterone is bound to a protein carrier A. B. C. D. E.



3.



Sex hormone binding globulin (SHBG) Albumin Corticosteroid binding globulin A and B Not all above



The major androgen products of the ovary are A. Testosterone B.Androstenedione C.Dehydroepiandrosterone (DHA) D. B and C E. A, B, C are true



4.



The excess genetic material of oocytes during each meiosis is extruded as A. B. C. D. E.



5.



The cells that produce GnRH originate from A. B. C. D. E.



6.



Estriol Progesterone Inhibin-A Androgen Estradiol



The progressive sequence (with steps omitted) in the metabolism of steroid hormone is A. B. C. D. E.



8.



Optic area Olfactory area Oculatory area A and B Not all above



In the normal menstrual cycle corpus luteum secretes the following hormones, except A. B. C. D. E.



7.



Cumulus oophorus Barr’s body Call-Exner body Cytoplasm Polar body



Cholesterol-estriol-testosterone-pregnenolone Cholesterol-pregnenolone-cortisol-estradiol Cholesterol-pregnenolone-estrone-androstenedione Cholesterol-androstenedione- pregnenolone-estrone Cholesterol-pregnenolone-androstenedione-estrone



Which of the following embryonic tissues contributes to the adult fallopian tubes? A. B.



Coeloemic epithelium Mesenchyme



C. D. E.



9.



Mesonephric duct Paramesonephric duct Urogenital ridge



Which of the following pubertal events is not mediated by gonadal estrogen production A. B. C. D. E.



10.



Breast development Menstruation Pubic hair growth Skeletal growth Vaginal cornification



Which of the following hormone(s) is / are produced by the corpus luteum ? A. B. C. D. E.



11.



Steroid hormones are classified as: A. B. C. D. E.



12.



Amino acids Phospholipids Lipids Glycoprotein None of the above



Management of anovulatory bleeding (dysfunctional uterine bleeding) depends on the following conditions, except A. B. C. D. E.



13.



Progesterone only Progesterone and estrogen only Progesterone, estrogen, and inhibin only Progesterone, estrogen, inhibin and relaxin only. Progesterone, estrogen, inhibin, relaxin and contractin.



Age of the patient Desire for fertility Size of the cervical os Amount of bleeding Cause of bleeding



Which of the following provide evidence of ovulation? A. Day 14 progesterone level B. C. D. E.



14.



Endometrial biopsy Normal menstrual cycle. Estrogen / progestogen ratio LH / FSH ratio



Which of the following is not used to correct anovulation? A. B. C. D.



Bromocriptine Weight loss hCG injection Low-dose estrogen



15.



16.



E. Clomiphene citrate One statement below is associated with prolactin A. Secreted by pituitary B. Its secretion suppressed by TRH C. Its secretion decrease at the time of lactation D. Its secretion induced by light E. Also secreted by placenta



One of the statements below is true about Mayer-Rokitansky-KusterHauser syndrome, A. B. C. D. E.



17.



Lack of mullerian development Presence of breast growth disturbance Occurs dysfunctional uterine bleeding Presence of sex hair growth disturbance Demonstrate augmented gonadotropin secretion during sleep



Phytoestrogens are classified into A. B. C. D. E.



Black cohosh, genistein, red clover Isoflavones, coumestans, and lignans Bonestein, black cohosh, lignans Isoflavones, red clover, bonestein Not all above



18.



Which one is the best predictor for the ovarian reserve ? A. B. C. D. E.



19.



Basal FSH Basal estradiol Anti mullerian hormone Inhibin B All of above



Which of the following is consistent with a diagnosis of delayed puberty? A. Breast budding in a 10-year-old girl B. Menarche delayed beyond 16 years of age C. Menarche 1 year after breast budding D. FSH values less than 20 mIU/mL E. Obese



20.



Normal stature with minimal or absent pubertal development may be seen in A. Testicular feminization B. Kallman syndrome C. Pure gonadal dysgenesis



D. Turner syndrome E. Intermittent athletic training



21.



Delayed puberty and sexual infantilism associated with hypergonadotropic can be seen in patients with which of the following?



hypogonadism



A. Adrenogenital syndrome (testicular feminization) B. McCune-Albright syndrome C. Kallman syndrome D. Gonadal dysgenesis E. Mullerian agenesis 22.



Which of the following pubertal events in girls is not estrogen dependent? A. Menses B. Vaginal cornification C. Hair growth D. Reaching adult height E. Production of cervical mucus



23.



Which of the following findings characterizes a normal semen sample? A. Agglutination B. Sperm concentration of 35 million per mL C. 5% normal sperm morphology D. 10% progressive sperm motility E. A volume of 1 mL



24.



The dominant follicle is formed on day ..... of the menstrual cycle A. B. C. D. E. 1 – 2



25.



12 – 14 9 – 11 5–7 2–4



The following statements is true regarding the short loop feed back of GnRH. A. Sex steroid relationship to pituitary. B. Conection of GnRH with it’s own secretion.



C. Conection of hypothalamus to pituitary D. With the down regulation mechanism E. Through feed back of hypophyseal portal system.



26.



The following is true regarding GnRH A. B. C. D. E.



27.



A patient with PCOS often have an increase in insuline resistence. This will result in an increase in A. B. C. D. E.



28.



FSH stimulates follicle growth only in the early preanthral stage. FSH increases its own receptor number on theca cells. FSH induces theca cell aromatase. FSH stimulates granulosa cell androgen production FSH induces granulosa cell LH receptors within the dominat follicle.



During the evaluation for infertility, a woman may have an endomertial biopsy to evaluate the quality of her ovulation since the development of the corpus luteum is most closely associated with A. B. C. D. E.



31.



Coelemic epithelium Mesenchyme Mesonephric duct Paramesonephric duct Urogenital ridge.



Which of the following statements best discribes the role of FSH in menstruation? A. B. C. D. E.



30.



FSH. Free estrogen level Free testosterone level Hepatic production of SHBG. Progestogen levels.



Which of the following embryonic tissues contributes to the adult fallopian tubes? A. B. C. D. E.



29.



The secretion is stimulated by norephineprin Hypothyroid will stimulate it’s secretion. Longterm and high dose of GnRH will increase multiple ovulation. Pineal gland hormone stimulate it’s secretion Oxytocin will increase it’s degradation.



Fertilization of an ovum Follicular phase of the endometrium Proliferative phase of the endometrium Secretory phase of the endometrium Sheding phase of the endometrium.



The three principal estrogens in women in decreasing order of potency are:



A. B. C. D. E.



32.



Which of the following is not used to correct anovulation? A. B. C. D. E.



33.



36.



37.



Sex steroid relationship to pituitary. Conection of GnRH with it’s own secretion. Conection of hypothalamus to pituitary With the down regulation mechanism Through feed back of hypophyseal portal system.



The following is true regarding GnRH: A. It’s a steroid hormone B. It has membrane receptor C. It enters posterior lobe of pituitary D. It directly enters from tractus tubero infundibular. E. The secretion is stimulated by dopamine. The following is true regarding early relationship of hypothalamus-pituitary-ovarian axis A. The first initiation is prior to puberty B. Initiated with FSH increase. C. At puberty, central hypersensitivity to estrogen is decline. D. Soon will lead to regular ovulation. E. Resulting in the first folliculogenesis.



The following is true regarding oocytes reserve A. B. C. D. E.



38.



It’s secreted by nucleus arcuatus It stimulates oxytocn secretion It stimulates GnRH secretion The secretion is stimulated by TRH It stimulates Norephineprin secretion



The following statements is true regarding the short loop feed back of GnRH. A. B. C. D. E.



35.



Bromocriptine Weight loss HCG injection Low-dose oestrogen Clomiphen citrate



The following statements regarding prolactin is true : A. B. C. D. E.



34.



Estradiol, estriol, estrone Estradiol, estrone, estriol Estriol, estradiol, estrone Estriol, estrone, estradiol Estrone, estriol, estradiol



The highest oocytes reserve is at the time of parturition. The development of primordial follicle disappear for sometime after parturition. At menarche there are 6 to 7 million oocytes. Pregnancy will influence the decline of oocytes reserve. Stimulation drugs inhance the decline of oocytes reserve.



The following is true regarding spermiogenesis (E) A. The development of diploid to haploid



B. C. D. E.



39.



The following is true regarding FSH in early cycle (C) A. B. C. D. E.



40.



Process of spermatogonia to primary spermatocyte. Process of primary oocyte to secondary spermatocyte. Process of secondary spermatocyte to spermatide. Maturation process of spematide to spermatozoa.



Stimulates IGF-1 secretion Stimulates IGF-2 secretion Stimulates inhibin secretion Inhibits activin secretion Stimulate androgen synthesis.



The following is true regarding follicle atresia in early cycle (A) A. B. C. D. E.



Androgenic micro environment is more dominant. Has more FSH receptors in the granulosa cells. The role of intra follicular activin is more dominant. It is not found in normal ovulatory cycle. It can disturb menstrual cycle.



41. In the normal menstrual cycle corpus luteum secretes the following hormones, except A. B. C. D. E.



42.



The following substances influence the amount of blood during menstruation, except : A. B. C. D. E.



43.



Oxytocin Matrix metallo proteinase (MMP) Tissue inhibitor metalloproteinase (TIMP) Transforming growth factor (TGF)-β Plasminogen activator inhibitor-1



In the normal menstrual cycle, bleeding stop due to the following, except A. B. C. D. E.



44.



Estriol Progesterone Inhibin-A Androgen Estradiol



The total shedding of endometrium Blood clotting factors Uterine contraction Vascular vasoconstriction. Prostaglandins



The following hormones have intracellular receptor, except (C) A. B. C. D.



Estradiol Progesterone FSH Testosterone



E. Androstenedion.



45.



The following is true regarding hCG (A) A. B. C. D. E.



46.



A patient with PCOS often have an increase in insuline resistence. This will result in an increase in A. B. C. D. E.



47.



ACTH Indomethacin Progesterone Prolactin-inhibiting-factor. Thyroid hormone.



Menarche usually occurs between the ages of (B) A. B. C. D. E.



50.



Arachidonic acid Isobutyric acid Isoleucocine Linoleic acid Phospholipase A



Given that prostaglandins appear to be involved in preterm labor, which of the following medications might provide some help in stopping preterm labor (B) A. B. C. D. E.



49.



FSH. Free estrogen level Free testosterone level Hepatic production of SHBG. Progestogen levels.



Which of the following substances is the precusor to protaglandins (PGs) (A) A. B. C. D. E.



48.



α- subunit hCG is the same with LH Secreted by corpus luteum It’s secretion is initiated at the time of fertilization. The peak concentration is in term pregnancy In ectopic pregnancy it’s concentration is higher than in normal pregnancy.



8 and 10 years 11 and 13 years 14 and 16 years 17 and 18 years Any time.



In addition of ethynil group at the 17C position of estradiol was critical in the development of the oral contraceptive pill because it (E) A. decreases biological activity B. Increases androgenic activity C. Increases hepatic degradation D. Increases SHBG affinity E. Maintain biological activity after oral absorbsion



A.



Select the one that is the best of the items



1. Prof. Dr. Samsulhadi SpOG(K) (Pembuat soal no.1 – 20)



19. The following statements regarding prolactin is true : (D) A. It’s secreted by nucleus arcuatus B. It stimulates oxytocn secretion C. It stimulates GnRH secretion D. The secretion is stimulated by TRH E. It stimulates Norephineprin secretion 20. The following statements is true regarding the short loop feed back of GnRH. (E) A. Sex steroid relationship to pituitary. B. Conection of GnRH with it’s own secretion. C. Conection of hypothalamus to pituitary D. With the down regulation mechanism E. Through feed back of hypophyseal portal system. 21. The following is true regarding GnRH: (B) A. It’s a steroid hormone B. It has membrane receptor C. It enters posterior lobe of pituitary D. It directly enters from tractus tubero infundibular. E. The secretion is stimulated by dopamine 22. The following is true regarding GnRH (A) A. The secretion is stimulated by norephineprin B. Hypothyroid will stimulate it’s secretion. C. Longterm and high dose of GnRH will increase multiple ovulation. D. Pineal gland hormone stimulate it’s secretion E. Oxytocin will increase it’s degradation. 23. The following is true regarding the secretion of GnRH (C) A. The amplitudo is higher in early cycle than in the luteal phase. B. In early cycle the amplitudo is higher than in the midcycle. C. In midcycle, the frequency of secretion is increase D. In midcycle the amplitudo is higher than in the luteal phase. E. In luteal phase the frequency is higher than in the early cycle.



24. The following is true regarding early relationship of hypothalamus-pituitaryovarian axis C)



A. B. C. D. E.



The first initiation is prior to puberty Initiated with FSH increase. At puberty, central hypersensitivity to estrogen is decline. Soon will lead to regular ovulation. Resulting in the first folliculogenesis.



25. The following is true regarding oocytes reserve (B) A. The highest oocytes reserve is at the time of parturition. B. The development of primordial follicle disappear for sometime after parturition. C. At menarche there are 6 to 7 million oocytes. D. Pregnancy will influence the decline of oocytes reserve. E. Stimulation drugs inhance the decline of oocytes reserve. 26. The following is true regarding oogenesis (meiosis) ( D). A. Oogonia has haploid chromosom B. The first polar body (polar body I) disconnected at the early stage of meiosis I C. Polar body II disconnected at the time of ovulation D. Meiosis disappear during follicular phase E. The final result is 4 oocytes 27. The following is true regarding spermiogenesis (E) A. The development of diploid to haploid B. Process of spermatogonia to primary spermatocyte. C. Process of primary oocyte to secondary spermatocyte. D. Process of secondary spermatocyte to spermatide. E. Maturation process of spematide to spermatozoa. 28. The following is true regarding the situation of early cycle (E) A. The LH receptor is in granulosa cells B. FSH stimulates the expression of of LH receptor. C. Estradiol and FSH stimulate LH receptor. D. Progesteron stimulates the secretion center of gonadotropin E. Estradiol inhibits the secretion center of gonadotropin 29. The following is true regarding FSH in early cycle (C) A. Stimulates IGF-1 secretion B. Stimulates IGF-2 secretion C. Stimulates inhibin secretion D. Inhibits activin secretion E. Stimulate androgen synthesis.



30. The following is true regarding follicle atresia in early cycle (A) A. Androgenic micro environment is more dominant. B. Has more FSH receptors in the granulosa cells.



C. The role of intra follicular activin is more dominant. D. It is not found in normal ovulatory cycle. E. It can disturb menstrual cycle. 31. The dominant follicle is primarily supported by the concentration of ( D) A. Progesterone B. IGF-1 C. IGF-2 D. Inhibin-B E. Activin 32. The dominant follicle is formed on day ..... of the menstrual cycle ( C) A. 12 – 14 B. 9 – 11 C. 5 – 7 D. 2 – 4 E. 1 – 2 33. In the normal menstrual cycle corpus luteum secretes the following hormones, except (A) A. Estriol B. Progesterone C. Inhibin-A D. Androgen E. Estradiol 34. At the time of LH surge, the following statements are true, except :( A) A. Progesterone secretion is sharply increase. B. LH surge happened. C. Granulosa cells secrete prostaglandins D. High concentration of estradiol serum E. Oocyte maturation inhibitor (OMI) destroyed. 35. The following substances influence the amount of blood during menstruation, except : (A) A. Oxytocin B. Matrix metallo proteinase (MMP) C. Tissue inhibitor metalloproteinase (TIMP) D. Transforming growth factor (TGF)-β E. Plasminogen activator inhibitor-1



36. In the normal menstrual cycle, bleeding stop due to the following, except ( C)



A. B. C. D. E.



The total shedding of endometrium Blood clotting factors Uterine contraction Vascular vasoconstriction. Prostaglandins



37. The following hormones have intracellular receptor, except (C) A. Estradiol B. Progesterone C. FSH D. Testosterone E. Androstenedion. 38. The following is true regarding hCG (A) A. α- subunit hCG is the same with LH B. Secreted by corpus luteum C. It’s secretion is initiated at the time of fertilization. D. The peak concentration is in term pregnancy E. In ectopic pregnancy it’s concentration is higher than in normal pregnancy.



2.Prof..Dr.Djaswadi Dasuki , PhD, SpOG(K) (Pembuat soal no.21- 40)



39. Gonadotropin releasing hormone (GnRH) stimulates the release of (C): A. Adrenocorticotropic hormone (ACTH) B. Growth hormone (GH) C. Luteinizing hormone (LH) D. Opiate peptide E. Thyroid-stimulating hormone. 40. Since the work up of an elevated prolactin level can be expensive, it is appropriate to draw prolactin levels: (C) A. Decrease shortly after sleep. B. Increase after ingesting high-carbohydrate meals. C. Increase during stress D. Decrease during surgery E. Decrease after excercise.



41. Low-density lipoprotein (LDL) cholesterol serves as the principal substrate for steroidogenesis. Which of the following statements regarding circulating LDL correct ? (C) A. LDL is formed after addition of triglyceride to very low-density lipoprotein (VLDL). B. LDL levels are negatively correlated with cardiovascular disease. C. LDL is the major carrier of cholesterol in the plasma. D. LDL enters the cells by passive diffusion. E. LDL facilitates the transport of polar lipids in the blood plasma. 42. The progressive sequence (with steps omitted) in the metabolism of steroid hormone is (E) A. Cholesterol-estriol-testosterone-pregnennolone. B. Cholesterol-pregnenolone- cortisol-estradiol. C. Cholesterol-pregnenolone-estrone-androstenedione. D. Cholesterol-androstenedione- pregnenolone- estrone. E. Cholesterol-pregnenolone-androstenedione-estrone. 43. In addition of ethinyl group at the 17C position of estradiol was critical in the development of the oral contraceptive pill because it (E) A. Decreases biological activity B. Increases androgenic activity C. Increases hepatic degradation D. Increases SHBG affinity E. Maitains biological activity after oral absorbtion. 44. A patient with PCOS often have an increase in insuline resistence. This will result in an increase in (C) A. FSH. B. Free estrogen level C. Free testosterone level D. Hepatic production of SHBG. E. Progestogen levels. 45. Which of the following substances is the precusor to protaglandins (PGs) (A) A. Arachidonic acid B. Isobutyric acid C. Isoleucocine D. Linoleic acid E. Phospholipase A



46. Given that prostaglandins appear to be involved in preterm labor, which of the following medications might provide some help in stopping preterm labor (B) A. ACTH B. Indomethacin



C. Progesterone D. Prolactin-inhibiting-factor. E. Thyroid hormone. 47. Which of the following embryonic tissues contributes to the adult fallopian tubes? (D) A. Coelemic epithelium B. Mesenchyme C. Mesonephric duct D. Paramesonephric duct E. Urogenital ridge. 48. The most common ovarian lession associated with excessive estrogen stimulation infants is a (D) A. Granulosa cell tumor B. Leiomyoma C. Serious cysadenoma D. Single large follicular cyst E. Theca cell tumor. 49. Menarche usually occurs between the ages of (B) A. 8 and 10 years B. 11 and 13 years C. 14 and 16 years D. 17 and 18 years E. Any time. 50. Which of the following pubertal events is not mediated by gonadal estrogen production (C). A. Breast development B. Menstruation C. Pubic hair growth D. Skeletal growth E. Vaginal cornification



51. Which of the following statements best discribes the role of FSH in menstruation? (E) A. FSH stimulates follicle growth only in the early preanthral stage. B. FSH increases its own receptor number on theca cells. C. FSH induces theca cell aromatase.



D. FSH stimulates granulosa cell androgen production E. FSH induces granulosa cell LH receptors within the dominat follicle. 52. The midcycle LH surge (E) A. Enhances thecal cell androgen production B. Luteinizes granulosa cells. C. Initiates resumption of meiosis D. Facilitates oocyte expulsion E. All of the above. 53. Which of the following gametes is released from the graafian follicle during ovulation ? (D) A. Primary oocyte B. Primary oocyte and first polar body C. Secondary oocyte D. Secondary oocyte and first polar body E. Secondary oocyte and second polar body. 54. Which of the following hormone(s) is/are produced by the corpus luteum ? (D) A. Progestogen only B. Progestogen and estrogen only C. Progestogen, estrogen, and inhibin only D. Progestogen, estrogen, inhibin and relaxin only. E. Progestogen, estrogen, inhibin, relaxin and contractin. 55. During the evaluation for infertility, a woman may have an endomertial biopsy to evaluate the quality of her ovulation since the development of the corpus luteum is most closely associated with (D) A. Fertilization of an ovum B. Follicular phase of the endometrium C. Proliferative phase of the endometrium D. Secretory phase of the endometrium E. Sheding phase of the endometrium.



56. The post coital test used in an infertility evaluation assesses the cervical mucus for ferning. The presence of ferning depends on which of the following hormones ? (A) A. Estrogen B. Estrogen and progesterone



C. hCG D. LH E. Progesteron. 57. Spinnbarkeit describes the (C) A. Amount of cervical mucus B. Clarity of cervical mucus C. Elasticity of cervical mucus D. Ferning of cervical mucus E. Viscocity of crvical mucus. 58. Menstrual flow is associated with the (E) A. Prolonged maintenence of estrogen B. Prolonged maintenence of progesterone C. Withdrawal of FSH D. Withdrawal of LH E. Withdrawal of progesterone.



3. Dr.Zain Alkaff SpOG(K) (Pembuat soal no.41 – 60 )



59. Even after menopause most women have circulating estrogen. It mainly originates from the aromatization of (C) A. Androstenedione to estrone by ovarian granulosa cells B. Androstenedione to estrone by ovarian theca cells C. Androstenedione to estrone by adipose tissue D. Testosterone to estradiol by adipose tissue tissue E. Estradiol to estrone by adipose tissue. 60. A. 50-year-old woman presents to her health care provider complaining of hot flaushes. Holt flushes entail (A) A. Peripheral redistribution of blood flow leading to sweating and elevated heart rate. B. Peripheral vasodilatation reflecting an increase in core body temperature. C. Subjective symptoms and always accompanying objective signs of vasomotor instability. D. Peripheral vasodilatation resulting from a direct LH action on sympathetic neurons E. An avarage duration of about 30 minutes. 61. With the osteoporosis the accelerated bone loss occurs 1 to 8 years after menopause (B). A. Causes an elevation in circulating parathyroid hormone levels B. Causes increased urinary loss of phosphorus and hydroxyproline



C. Does not influence trabecular bone D. Primarily affects cortical bone. E. All of above. 62. Cardioprotective actions of estrogen may include (B) A. Depression of HDL levels. B. Dilatation of coronary vesels C. Elevation of LDL levels D. Production of thromboxane E. Widening of pulse pressure. 63. Which of the following substances stimulates GnRH secretion? (D) A. Beta-endorphin B. Dopamine C. Dynorphin D. Norepinephrine E. Serotonin 64. GnRH-associated peptide (GAP) is a/an. (A) A. Stimulator of gonadotropins and an inhibitor of prolactin B. Inhibitor of gonadotropins and a stimulator of prolactin C. Stimulator of gonadotropins and a stimulator of prolactin D. Inhibitor of gonadotropins and an inhibitor of prolactin E. Stimulator of gonadotropins and has no effect on prolactin 65. Tyrosine is an essential amino acid for the biosynthesis of (A) A. Dopamine B. Dynorphin C. GnRH D. Prostaglandins E. Vasopressin 66. A patient presents with amenorrhoea and galactorrhoea. Her prolactin levels are elevated. She is not and never has been pregnant. In additional to evaluating her for a prolactinoma, one also needs to evaluate for other causes that would increase prolactin such as elevated. (E) A. Corticotrophin-releasing hormone B. Dopamine C. Gamma-aminobutyric acid (GABA) D. Histamine type II receptor activation E. Thyrotropin-releasing hormone (TRH) 67. During which of the following conditions would the serum prolactin level be greatest? (C) A. Menopause B. Ovulation C. Parturition



D. Sleep E. Rest



68. Which of the following statements regarding GnRH-stimulated LH secretion is accurate? (C) A. It is enchanced by gonadotrope exposure to progesterone B. It is increased by gonadotrope exposure to testosterone C. It is enchanced by gonadotrope exposure to estrogen D. It is enchanced by gonadotrope exposure to continous GnRH E. It is associated with steady LH release 69. Steroid hormones are classified as: (C) A. Amino acids B. Phospholipids C. Lipids D. Glycoprotein E. None of the above 70. Adrenal corticoids belong to which of the following groups? (E) A. Aldehydes B. Androstanes C. Cholesterols D. Estranes E. Pregnanes 71. The three principal estrogens in women in decreasing order of potency are: (B) A. Estradiol, estriol, estrone B. Estradiol, estrone, estriol C. Estriol, estradiol, estrone D. Estriol, estrone, estradiol E. Estrone, estriol, estradiol 72. The following effects is attributed to hCG, except: (E) A. Maintenance of corpus luteum during early pregnancy B. Regulation of fetal adrenal androgen production C. Regulation of fetal testicular androgen production D. Stimulation of thyroid activity E. Inhibition of peripheral glucose uptake



73. The following substances are placental protein hormones, except: (A) A. Dopamine B. Inhibin C. GnRH



D. Human chorionic thyrotropin (hCT) E. Thyroid-releasing hormone (TRH) 74. The following statements regarding lactation are TRUE, except: (D) A. Estrogen stimulates mammary ductal proliferation B. Progesterone stimulates mammary alveolar development C. High circulating sex steroid levels during pregnancy inhibit lactation D. The gestational mammary gland produces a transudate containing casein and alpha-lactalbumin E. The postpartum decline in circulating sex steroid levels initiates lactation 75. Management of anovulatory bleeding (dysfunctional uterine bleeding) depends on the following conditions, except: (C) A. Age of the patient B. Desire for fertility C. Size of the cervical os D. Amount of bleeding E. Cause of bleeding 76. Treatment of hyperprolactinemia results in the following, except: (B) A. Elimination of galactorrhoea B. Prevention of acne C. Establishment of normal estrogen production D. Treatment of prolactin-secreting pituitary adenomas E. Induction of ovulation 77. The following statements regarding PCOS are true, except: (D) A. Elevated androgen production decreases sex hormone-binding globulin (SHBG) synthesis B. Elevated androgen production increases peripheral aromatization C. Acyclic estrogen production increases LH secretion D. Acyclic estrogen production decreases bone mineralization E. Acyclic estrogen production increases endometrial proliferation 78. Which of the following is manifestation of virilization? (B) A. Intermammary hair B. Clitoromegaly C. Abdominal hair D. Hair on upper legs E. Deep voice 4.Dr.Amino Rahardjo SpOG (K) (Pembuat soal no.61 – 80 )



79. Which of the following is the principal site of production of sex hormonebinding globulin (SHBG)? (C) A. Kidney B. Pituitary C. Liver D. Hypothalamus E. None of above 80. Which of the following medications is most likely to cause hirsutism? (A) A. Danazol B. Cimetidine C. Tetracycline D. Α-methyldopa E. Diazepam



81. Which of the following is the most common cause of pathologic hirsutism? (B) A. Congenital adrenal hyperplasia B. Polycystic ovary syndrome C. Cushing’s syndrome D. Adrenal tumor E. Turner syndrome 82. Which layer of the endometrium is refractory to progesterone? (C) A. stratum compactum B. stratum functionalis C. stratum basalis D. stratum spongiosum E. All above 83. Endometrial biopsy is indicated in all of the following situations except: (A) A. all women over age 35 B. women over age 35 with metrorrhagia C. menorrhagia not responding to medical therapy D. women at high risk for endometrial adenocarcinoma with dysfunctional uterine bleeding (DUB) E. women taking unopposed estrogens with DUB 84. In perimenopausal women, the following is true of the follicular and luteal phase: (D) A. Both phases lengthens B. The follicular phase shortens and the luteal phase lengthens C. The luteal phase shothens and the follicular phase lengthens D. Both phases shorthen E. Both phases are still in normal cycle. 85. Initial workup for a patient with postmenopausal bleeding should include all of the following except: (D) A. Pap smear



B. C. D. E.



Pelvic exam Rectal exam Office hysteroscopy Endometrial sampling



86. Adenomyosis is defined as: (B) A. Adenosis of the intramural myometrium B. Ectopic endometrial tissue extending more than 2 low power fields (LPFs) deeper than the basalis layer into the myometrium C. Dysfunctional uterine muscle that does not contract following endometrial shedding D. Endometrium growing at least 2 high power fields (HPFs) into the intramural layer of the uterus E. Endometrium growing less than 2 high power fields (HPFs) into the intramural layer of the uterus 87. Which of the following drugs are useful in the treatment of menorrhagia? (D) A. Cabergoline B. Oil of evening primrose C. Diazoxide D. Naproxen E. Diazepam. 88. Which is the following statements is not true? (D) A. Pulsatile secretion of the GnRH controls the synthesis of FSH and LH B. Prostaglandin-induced constriction of the spiral arterioles causes endometrial ischaemia C. Progesterone causes decidualisation of the endometrium D. Secretion of prolactin from the anterior pituitary is needed to maintain a regular cycle E. Folliculogenesis is under FSH control. 89. Regarding hirsutism, which one is not true (E) A. SHBG is usually decreased B. It may be successfully treated with the combined oral contraceptive pill C. In the presence of a regular menstrual cycle it is usually idiophatic D. It’s commonly related with PCOS E. It may be caused by diazepam 90. Regarding endometriosis: (A) A. Asymptomatic deposits of endometriosis are found in 20% of women of reproductive age B. Once found it should always be treated C. May not present with a painful nodule in a surgical scar D. It’s always persists until the menopause. E. It can be treated totally with GnRH analog.



91. Regarding the normal menstrual cycle: (D) A. FSH promotes luteal phase development B. Progestogen is produced by granulose cells. C. Oestrogen is produced mainly by theca cells D. Prostaglandins stimulate myometrial contraction. E. Progesterone peaks in the follicular phase 92. Which of the following may cause pelvic pain? (B) A. Retroverted mobile uterus B. Ovarian hyperstimulation C. Primary syphilis D. Policystic ovary syndrome E. Endometrial polyp. 93. Regarding the polycystic ovary syndrome: (E) A. SHBG levels are increased B. Androgens are decreased C. LH secretion is decreased D. Is not related with infertility E. Menstrual irregularity is a common feature 94. Which one (D) A. B. C. D. E.



is not include in the side effects of treatment for endometriosis: Acne Hot flushes Decrease in breast size Endometrial hyperplasia Weight gain



95. Which of the following statements is not true? (E) A. Removal of both ovaries cures premenstrual syndrome B. Visceral pain from the uterus, fallopian tubes and ovaries is transmitted via the autonomic nervous system (T10-L1) C. Cone biopsy of the cervix can cause secondary dysmenorrhoea D. Marker of Ca-125 can be used as diagnostic tool of endometriosis. E. Endometriosis related with cancer. 96. Which of the following is not true regarding the IUD? (C) A. Serum copper levels are unchanged B. The levonorgestrel-containing IUD is associated with a small increase in the haemoglobin level C. Pregnancy with IUD insitu should be terminated. D. The copper-containing IUD is associated with a small decreased in the haemoglobin level E. The plastic frame of the levonorgestrel-containing IUD contains barium so that it can be seen on X-ray



97. Which of the following is not caused by endometriosis? (D) A. Haemoptysis B. Infertility C. Renal failure D. Cancer E. Dispareunia.



5. Prof.Dr.Mochamad Anwar M.Med.Sc, SpOG(K)



98. Which of the following statements regarding the POP (progestogen only pill) are true? (E) A. It works mainly by inhibiting ovulation B. The dose of progestogen is higher than that in the combined pill C. It does not appear in the breast milk D. It does not influence motility of sperm E. It is suitable for both diabetic and hypertensive women 99. Regarding the combined oral contraceptive (COC) pill: (B) A. The incidence of iron deficiency anaemia is slightly increase B. The COC is protective against both ovarian and endometrial cancer C. It does not carries an increased risk of venous thromboembolism D. The incidence of the benign breast disease is increased by 30% E. It is not suitable for young women for postponing pregnancy. 100. Which of the following is not true? (B) A. Injectable progestogens inhibit lactation B. Post-coital IUD insertion is effective if fitted within 7 days of unprotected intercourse C. The injectable progestogen depo-provera inhibits ovulation D. Return of fertility may be delayed following removal of a contraceptive implant E. Implant contraceptive method is suitable for post partum women. 101. Regarding contraception and the menopause: (A) A. The low-dose COC can be used up to the menopause in healthy non-smoking women B. Measurement of FSH is an accurate indicator of fertility C. HRT can be used as contraception D. Sterilization affects the timing of menopause E. For pre-menopausal women, serum FSH concentration is not considered for taking COC.



102. Regarding the post-coital test: (D) A. It must be done on day 21 of the cycle B. It can detect anovulation C. It can detect tubal problems D. It can detect azoospermia E. It can detect acrosome reaction of sperm. 103. Which of the following provide evidence of ovulation? (B) A. Day 14 progesterone level F. G. H. I.



Endometrial biopsy Normal menstrual cycle. Estrogen / progestogen ratio LH / FSH ratio



104. Which of the following is not used to correct anovulation? (D) A. Bromocriptine B. Weight loss C. HCG injection D. Low-dose oestrogen E. Clomiphen citrate 105. Which of the following may not cause male infertility? (D) A. Gonorrhoea infection B. Cystic fibrosis C. Prolactinoma D. Hydrocele E. Varicocele 106. Which of the following are associated with early pregnancy loss? (E) A. Sexual intercourse B. Multiple partner C. Wart virus infection D. Pulmonal tuberculosis E. Intra-uterine pregnancy with an intra-uterine device in situ



107. Regarding osteoporosis, which one is not true: (D) A. Hip fracture causes more deaths than cancer of the cervix, uterus and ovaries combined B. Bone densitometry can be used to screen individual at risk C. May be associated with hypogonadism D. Fracture is more common on stratum compactum instead of trabecular.



E. Osteocyt : osteoblasts that are trapped in cortical bone during the remodeling process. 108. Which the following are associated with an increased risk of osteoporosis? (A) A. Malabsorption syndrome B. Steroid therapy C. Hyperparathyroidism D. Late menopause E. Premature ovarian failure (POF) 91. Case-1 A 32-year-old nulliparous woman has had oligomenorrhea since menarche. During the past 5 years she has experienced slow but progressive increase in hair on her face, back and forearms in the inter-mammary space, and on the back of her hands. Her voice has slowly deepened and temporal balding and clitoromegaly have developed. Which of the following is the most likely diagnosis? (C) A. Polycystic ovary syndrome B. C. D. E.



Cushing’s syndrome Stromal hyperthecosis Ovarian tumor Asherman’s syndrome



92. Case-2. A couple have a 2-year history of infertility. The male partner is healthy with no past serious illnesses, and has one child from his previous marriage. The female partner has a regular 29-day menstrual cycle and has a normal body mass index. Her only previous pregnancy was an ectopione. Which of the following investigations is most likely to reveal the cause of the infertility? (E) A. B. C. D. E.



Semen analysis Pelvic ultrasound examination Hormonal profile Hysteroscopy Laparoscopy



Case-4 (questions 93 – 95) A 47-year-old patient presents wondering if her problems with mood, swings, insomnia, and vaginal dryness represent menopause. She had a hysterectomy 10 years ago for abnormal uterine bleeding, but the uterus were not removed.



Since she cannot afford hormonal testing, a maturation index is done on her pap smear.



93. Ideally, cytologic cells for evaluation of hormonal status should be obtained from the (D) A. B. C. D. E.



Ectocervix Endocervix Labia minora Lateral vaginal wall Posterior vaginal fornix.



94. Ninety percent of the cells found on her pap smear have thick, rounded cytoplasm and plump, round, vesicular nuclei with an intact chromatin patter. The maturation index (MI) would most likely be (B) A. 90 / 0 / 10. B. 90 / 10 / 0 C. 10 / 0 / 90 D. 10 / 90 / 0. E. 0 / 90 / 10 95. Based on this result, one would anticipate that if hormonal levels were obtained they would show (C) A. B. C. D. E.



Elevated estrogen level Elevated progesterone levels. Low estrogen levels Normal estrogen levels Normal progesterone levels



Case-5 A patient presented to the emergency department with an infected incomplete abortion. During the dilatation and curettage (D&C) excessive bleeding developed that required vigorous curetting to control. She returns to the physician 6 months later complicating that she has not had a menstrual cycle. She has all the symptoms of getting ready to start a period but never sees any bleeding.



96. This history implies that what layer of endometrium is damaged ? (A) A. Basal zone B. C. D. E.



Compact zone Functional zone Spongy zone None of above



6. Prof.Dr.Samsulhadi SpOG (K) (Pembuat soal 97 – 100 )



Case-6. (for questions 97 – 100 ) Woman of 30 years, married for 5 years and no children, seeing the doctor for planning pregnancy. The menstruation has been lasting for 10 days with irregular period. The body height is 155 cm, body weight 80 kg, blood pressure 140/90, pulse rate 80/minute. Vaginal ultrasound examination: the size of uterus is normal, no gestational sac, endometrial thick 1.9 mm, enlargement of both ovaries with > 10 follicles of 7-8 mm diameter on a necklace type, stroma is thick. Two times sperm analysis is azoospema, FSH of husband is 6 mIU/ml.



97. Based on the Roterdam consensus 2003, for the diagnosis of PCOS on this case, is still needed to exam : (E) A. Serum estradiol B.Ratio LH/FSH C.Fasting insulin serum D. Total testosterone serum E.None of above. 98. The first step to overcome menstrual problems for this case should be treated with (D) A. GnRH agonist B. GNRH antagonist C. Aromatase inhibitor D. Progestogen E. Uterotonic 99. In the long term, this women will be in a high risk of (A) A. NID-DM B. Myometrial malignancy C. Ovarial malignancy D. Osteoporosis E. Alzeimer.



100. ART problems might be happened in this case (D) A. Low FSH serum concentration B. High estradiol serum concentration C. High androgen serum concentration D. High LH serum concentration E. Low progesterone serum concentration.



1. The differences between RNA and DNA are, except : a. b. c. d. e.



RNA uses pentose sugar RNA is single stranted RNA contain uracil Sugar in RNA is ribose RNA carries codon



What happen in increase Corticotrophin Releasing Hormone (CRH) in fetal development, All statement in bellow are true; except : A. Increase fetal cortisol is positive feedback relationship B. Augment fetal ACTH secretion. C. Adjunct progesterone secretion. D. More DHEAS. E. Increase estrogen



2. Which regard of the following is related to the first sign of pubertal development of female : a. Menarche b. Menopause c. Pubarche d. Menstruation e. Telarche 3. Mechanism of action for steroid hormones : A. Rapidly transported across the cell membrane by osmose action. B. All of which require direct interaction with DNA. C. Via hormone-receptor complex to bind hormone-responsive element in DNA. D. Not regulate posttranscriptional events. E. Translation via DNA methylation.



4. This statement is not true about basic mechanisms of menstruation a. b.



Menstruation is preceeded by intense vasoconstriction of the spiral arterioles Ischaemia induced by intense vasoconstriction of 4 to 24 hours will be augmented by increased release of free oxygen radicals produced by tissue (endometrial) desquamation c. Platelet fibrin plug formation is grossly deficient in endometrium



d.



Ruptured spiral arterioles do not form platelet fibrin thrombi in the early phases of menstruation e. The rapid formation of fibrin plugs and their degradation play a part of the control of menstrual bleeding 5.The purpose of the progestational challenge test is to asses the level of endogenous : A. Testosterone B. Progesterone C. Estrogen D. Prolactin E. TSH 6. Which of the following sequences best describes estrogen action? a. cell membrane diffusion, steroidreceptor,-DNA complex formation, transcription, translation b.cell membrane receptor activation, steroidreceotor DNA complex formation, translation,transcription, transcription c. cell membrane diffusion' steroidreceptor-DNA complex formation, translation, transcription d.cell membrane diffusion, adenylatecyclase activation, cAMP production, protein phosphorylation e. cell membrane receptor activation, adenylatecyclase activation, c AMP production, protein phosphorilation..



7.Amenorhea should be evaluated in any patient without a periode who has an absence of growth or development of secondary sexual characteristic by age A. 8 B. 10 C. 12 D. 14 E. 16 8.Sperm capacitation refers to a process by which spermatozoa become capable of: a. stimulating meiosis of ovum b. dispersing the zona radiata c. penetrating the cervical mucus d. producing acrosomal enzymes e. fertilizing the ovum



9.Gonadotropin releasing hormone (GnRH) stimulates the release of: a. b. c. d. e.



Adrenocorticotropic hormone (ACTH) Growth hormone (GH) Luteinizing hormone (LH) Opiate peptide Thyroid-stimulating hormone (TSH)



10. This statement is not true about basic mechanisms of menstruation a. Menstruation is preceeded by intense vasoconstriction of the spiral arterioles b. Ischaemia induced by intense vasoconstriction of 4 to 24 hours will be augmented by increased release of free oxygen radicals produced by tissue (endometrial) desquamation c. Platelet fibrin plug formation is grossly deficient in endometrium d. Ruptured spiral arterioles do not form platelet fibrin thrombi in the early phases of menstruation e. The rapid formation of fibrin plugs and their degradation play a part of the control of menstrual bleeding



11. Which of the following refers to WHO consensus 1999 about normosperm : a. b. c. d. e.



consentration less than 20 million/ml volume at least 2 ml grade A at least 50% grade A + B more than 2% leucocyte less than 2/ml



12.Regulation of fetal adrenal gland in relation with labor process, in late gestation : A. The mother prevent high cortisol by converting to cortisone. B. Increasing estogen maternal. C. Increasing 11β- hydroxysteroid dehydrogenase. D. A, B and C true. E. A and C true. 13.29-year-old primigravida who received no prenatal care has marked vaginal bleeding after the onset of labor at 38 weeks gestation. Cesarean section is performed and a lacerated low-lying placenta is removed. She remains hypotensive for 6 hours and requires transfusion of 12 packed RBC units. Postpartum, she becomes unable to breast-feed the infant. She does not have a resumption of normal menstrual cycles. She becomes more sluggish and tired. Laboratory findings include hyponatremia, hyperkalemia, and hypoglycemia. Which of the following pathologic lesions is she most likely to have had following delivery?







Bilateral adrenal hemorrhage







 Pituitary necrosis







 Subacute thyroiditis







 Metastatic choriocarcinoma







 Insulitis



14. A patient presents with amenorrhea and galactorrhea. Her prolactin level are elevated. She is not and never has been pregnant. In addition to evaluating her for a prolactinoma, one also needs to evaluate for other causes that would increase prolactin such as elevated. a. dopamine b. gamma-aminobutyric acid c. hystamin type II receptor activation d. Thyrotropin releasing hormone e. corticotrophin releasing hormone



15. The initial step in the workup of amenorheic patient after excluding pregnancy begin with measurement of : A. TSH B. LH C. AMH D. FSH E. GnRH 16. Which of the following sequences best describes estrogen action? a. cell membrane diffusion, steroidreceptor,-DNA complex formation, transcription, translation b. cell membrane receptor activation, steroidreceotor DNA complex formation, translation, transcription, transcription c. cell membrane diffusion' steroidreceptor-DNA complex formation, translation, transcription d. cell membrane diffusion, adenylatecyclase activation, cAMP production, protein phosphorylation e. cell membrane receptor activation, adenylatecyclase activation, c AMP production, protein phosphorilation..



17. Physical examination of amenorhea : A. Body mass index



B. C. D. E.



Breast development The presence of pubic hair growth Evaluation of the genital outflow and uterus If all are correct



18. Which of the following causes of delayed puberty accompanies elevated circulating gonadotropin levels? a. Kallman's syndrome b. Hypothalamic tumors c. gonadaldysgenesis d. malnutrition e. chronic illness



19. Which of the following sequences best describes estrogen action? a. cell membrane diffusion, steroidreceptor,-DNA complex formation, transcription, translation b. cell membrane receptor activation, steroidreceotor DNA complex formation, translation, transcription, transcription c. cell membrane diffusion' steroidreceptor-DNA complex formation, translation, transcription d. cell membrane diffusion, adenylatecyclase activation, cAMP production, protein phosphorylation e. cell membrane receptor activation, adenylatecyclase activation, c AMP production, protein phosphorilation..



20.What happen in increase Corticotrophin Releasing Hormone (CRH) in fetal development, All statement in bellow are true; except : A. Increase fetal cortisol is positive feedback relationship B. Augment fetal ACTH secretion. C. Adjunct progesterone secretion. D. More DHEAS. E. Increase estrogen



Compared to users of combination oral contraceptive, users of progestin only are likely to experience? A. Intrauterine pregnancy B. Irregular vaginal bleeding C. Gonadotropin suppression D. Ectopic pregnancies E. Mood swings 02. The primary mechanism by which oral contraceptive prevent pregnancy? A. Inhibiting serum FSH levels B. Inhibiting serum LH levels C. Inducing endometrial atrophy D. Inducing lymphatic endometritis E. Increasing cervical mucus viscosity 03. The primary mechanism of IUD is A. Creating chronic endometritis B. Preventing fertilization C. Inhibiting ovulation D. Altering tubal motility E. Destroying sperm 04. Which of the following surgical approaches for sterilization procedure is associated with the highest failure rate? A. Postpartum mini laparotomy B. Interval mini laparotomy C. Laparoscopy D. Hysteroscopy E. Vaginal colpotomy 05. Reducing the estrogen content of oral contraceptive results in an increase in the rate of? A. Pregnancy B. Breakthrough bleeding C. Thromboembolic complication D. Insulin resistance E. Premenstrual syndrome 06. The administration of RU 486 result in A. Abortion when givinen in early pregnancy B. Delayed menses when given during midluteal phase C. Menses when given during follicular phase D. Resistance to prostaglandin inhibitors



E. Induction of progesterone receptors in endometrium 07. A 31 year old infertility patient with regular ovulatory menstrual sycles has begun theraphy with clomiphene citrate. Before she starts theraphy, what information should you provide her regarding the medication? A. The timing of ovulation is increased by a a week B. Approximately 40% of patient will respond to clomiphene citrate with increased endometrial thickness C. The risk of multiple gestation is 25% D. CC improves the fecundity rate principally through its effect on the endometrial lining E. Risk and side effects of CC include nausea, hot flushes, weight gain and mood swings 08. The initial treatment of choice in a patient with hypogonadotric hypogonadism when ovulation is desired is A. Low dose estrogen theraphy B. Human menopausal gonadotropin (hMG) theraphy C. Bromocriptine mesylate D. Clomiphene citrate E. Cyclic progesterone 09. Which of the following statements best describes estrogen action on cervical mucus? A. It decreases the water content of cervical mucus B. It decreases the palm-leaf crystallization pattern of mucus upon drying C. It decreases formation of glycoprotein channels favoring sperm penetration D. It increases cervical mucus stretchability (spinkerbait) E. It increases the amount of potassium chloride in the cervical mucus 10. The highest rate complication is associated with second-trimester pregnancy termination using? A. Intravenous oxytocin B. Intravenous prostaglandin C. Intravaginal prostaglandin D. Intramuscular prostaglandin E. Dilation & evacuation 11. Which of the following statements regarding basal body temperature is true? A. An oral temperature is taken prior to bedtime B. a rise of 0,2°F between 2 consecutive days reflects ovulation C. a biphasic temperature shift reflects estrogen action on the hypothalamus D. absence of biphasic temperature shift suggest pregnancy E. none of the above



12. Which of the following is the best method to time intercourse for procreative means? A. Thermogenic shift in basal body temperature B. Urinary luteinizing hormone (LH) kit testing C. Serum progesterone level D. Profuse, thin, acellular cervical mucus E. Mittelschemerz 13. IUD s are associated with an increase in A. Salpingitis in the first few weeks of use B. Salpingitis in long term users C. The number of ectopic pregnancy D. Dysmenorrhea in multiparous patients E. The rate of PID



14. Which of the following statements regarding the postcoital test is true? A. It predicts whether pregnancy can occur B. It is performed 1 to 2 days after ovulation C. It correlates the number of sperm in the cervical mucus with the pregnancy rate D. It examines the ability of sperm to reach and survive in the mucus E. It is performed within 1 hour of coitus 15. Besides infertility, the most common symptoms of luteal phase defects is A. Vaginal dryness B. Spontaneous miscarriage C. Tubal occlusion D. Breast tenderness E. Ovarian enlargement 16. Which of the following causes for infertility may be treatable by assisted reproductive technology? A. Fallopian tube obstruction B. Low sperm count C. Cervical mucus abnormalities D. Unexplained infertility E. All of the above 17. Infertility from endometriosis may be due to which four of the following abnormalitie, except? A. Pelvic adhesions B. altered fallopian tube motility C. sperm phagocytosis by peritoneal macrophages D. defective embryo implantation E. pituitary failure 18. Which of the following hormones decreases after first trimester pregnancy? A. Progesterone B. Prolactin C. hcg D. Human placental lactogen E. Estriol 19. A 22 years old woman with amenorrhea of 6 weeks duration undergoes surgery for acute appendicitis. At the time of surgery, a 3 cm semisolid left ovarian cyst is discovered. It is vascular and appears to contain a blood filled central cavity. A serum pregnancy test is positive. Which of the following procedure should be done? A. Ovarian cystectomy B. Ovarian wedge resection C. Oophorectomy D. Salphingo-oophorectomy E. None of the above



20. A 16 year old girl has not experienced menarche. Examination shows absence of breast development and small but otherwise normal female pelvic organs. Which of the following diagnostic tests is most useful in determining the etiology of the amenorrhea? A. FSH B. Serum estradiol C. Serum testosterone D. MRI of the head E. Ovarian biopsy 21. A 28 year old patient complains of amenorrhea after D&C for postpartum bleeding. The most likely diagnosis is? A. Gonadal dysgenesis B. Shehaan syndrome C. Kallman syndrome D. Mayer-Rokitansky-Kuster-Hauser syndrome E. Asherman syndrome 22. Prolactin secreting pituitary adenomas usually? A. Diminish in size during pregnancy B. Increase in size over time C. Are symptomatic during lactation D. Impinge on the olfactory nerve E. Respond to medical therapy 23. Management of anovulatory bleeding (dysfunctional uterine bleeding) depends on which four following conditions, except? A. Age of the patient B. Desire for fertility C. Size of cervical os D. Amount of bleeding E. Cause of bleeding 24. Treatment of hyperprolactinemia results in which four of the following, except? A. Elimination of galactorrhea B. Prevention of acne C. Establishment of normal estrogen production D. Treatment of prolactin-secreting pituitary adenomas E. Induction of ovulation 25. Hirsutism is associated with which four of following conditions? A. PCOS B. CAH C. Cushing syndrome D. Increased androgen utilization by skin E. Hyperprolactinemia 26. Anovulatory bleeding (dysfunctional uterine bleeding) is? A. Common in prepubertal girls



B. C. D. E.



Common in the early teens Usually suggestive of steroid-production ovarian tumors Uncommon during the perimenopausal years Dependent on the presence of progesterone



27. An 18 year old woman comes to your clinic with irregular cycles since menarche and mild hirsutism. She is not interested in pregnancy or contraception. Her serum TSH, prolactin, and DHEAS levels are normal, with slightly elevated serum testosterone level of 80 ng/dL. Which of the following is the most appropriate next step for this patient? A. Oral contraceptive treatment B. Endometrial biopsy C. GnRH stimulation test D. Clomiphene citrate E. Bromocriptine 28. Which contraceptive method has the lowest pregnancy rate in 100 woman using the method for 1 year? A. IUD B. Long acting progestins C. Diaphragm D. Oral contraceptives E. Spermicidal cream 29. Spermicides destroy spermatozoa primarily by? A. Activating acrosomal enzymes B. Disrupting cell membranes C. Inhibiting glucose transport D. Altering vaginal enzymes E. Increasing vaginal pH 30. In a young, obese, chronically anovulatory woman with an elevated LH:FSH ratio and polycystic appearing ovaries, which of the following if the preferred initial method of ovulation induction? A. Metformin B. Human menopausal gonadotropins C. Pulsatile GnRH D. Clomiphene citrate E. Bromocriptine mesylate



A patient with PCOS often have an increase in insuline resistence. This will result in an increase in A. FSH. B. Free estrogen level C. Free testosterone level D. Hepatic production of SHBG. E. Progestogen levels. 2.



Which of the following substances is the precusor to protaglandins (PGs) A. Arachidonic acid B. Isobutyric acid C. Isoleucocine D. Linoleic acid E. Phospholipase A



3.



Given that prostaglandins appear to be involved in preterm labor, which of the following medications might provide some help in stopping preterm labor A. ACTH B. Indomethacin C. Progesterone D. Prolactin-inhibiting-factor. E. Thyroid hormone.



4.



During the evaluation for infertility, a woman may have an endomertial biopsy to evaluate the quality of her ovulation since the development of the corpus luteum is most closely associated with A. Fertilization of an ovum B. Follicular phase of the endometrium C. Proliferative phase of the endometrium D. Secretory phase of the endometrium E. Sheding phase of the endometrium.



5.



The post coital test used in an infertility evaluation assesses the cervical mucus for ferning. The presence of ferning depends on which of the following hormones ? A. Estrogen B. Estrogen and progesterone C. Hcg D. LH E. Progesteron.



6.



Management of anovulatory bleeding (dysfunctional uterine bleeding) depends on the following conditions, except: A. Age of the patient



B. C. D. E.



Desire for fertility Size of the cervical os Amount of bleeding Cause of bleeding



7.



Treatment of hyperprolactinemia results in the following, except: A. Elimination of galactorrhoea B. Prevention of acne C. Establishment of normal estrogen production D. Treatment of prolactin-secreting pituitary adenomas E. Induction of ovulation



8.



The following statements regarding PCOS are true, except: A. Elevated androgen production decreases sex hormone-binding globulin (SHBG) synthesis B. Elevated androgen production increases peripheral aromatization C. Acyclic estrogen production increases LH secretion D. Acyclic estrogen production decreases bone mineralization E. Acyclic estrogen production increases endometrial proliferation



9.



Which of the following is manifestation of virilization? A. Intermammary hair B. Clitoromegaly C. Abdominal hair D. Hair on upper legs E. Deep voice



10.



Endometrial biopsy is indicated in all of the following situations except: A. all women over age 35 B. women over age 35 with metrorrhagia C. menorrhagia not responding to medical therapy D. women at high risk for endometrial adenocarcinoma with dysfunctional uterine bleeding (DUB) E. women taking unopposed estrogens with DUB



11.



In perimenopausal women, the following is true of the follicular and luteal phase: A. Both phases lengthens B. The follicular phase shortens and the luteal phase lengthens C. The luteal phase shothens and the follicular phase lengthens D. Both phases shorthen E. Both phases are still in normal cycle.



12.



Initial workup for a patient with postmenopausal bleeding should include all of the following except: A. Pap smear B. Pelvic exam C. Rectal exam D. Office hysteroscopy E. Endometrial sampling



13.



Adenomyosis is defined as: (B) A. Adenosis of the intramural myometrium B. Ectopic endometrial tissue extending more than 2 low power fields (LPFs) deeper than the basalis layer into the myometrium C. Dysfunctional uterine muscle that does not contract following endometrial shedding D. Endometrium growing at least 2 high power fields (HPFs) into the intramural layer of the uterus E. Endometrium growing less than 2 high power fields (HPFs) into the intramural layer of the uterus



14.



Which of the following drugs are useful in the treatment of menorrhagia? A. Cabergoline B. Oil of evening primrose C. Diazoxide D. Naproxen E. Diazepam.



15.



Which is the following statements is not true? A. Pulsatile secretion of the GnRH controls the synthesis of FSH and LH B. Prostaglandin-induced constriction of the spiral arterioles causes endometrial ischaemia C. Progesterone causes decidualisation of the endometrium D. Secretion of prolactin from the anterior pituitary is needed to maintain a regular cycle E. Folliculogenesis is under FSH control.



16.



Regarding hirsutism, which one is not true A. SHBG is usually decreased B. It may be successfully treated with the combined oral contraceptive pill C. In the presence of a regular menstrual cycle it is usually idiophatic D. It’s commonly related with PCOS E. It may be caused by diazepam



17.



Regarding endometriosis: A. Asymptomatic deposits of endometriosis are found in 20% of women of reproductive age B. Once found it should always be treated C. May not present with a painful nodule in a surgical scar D. It’s always persists until the menopause. E. It can be treated totally with GnRH analog.



18.



Regarding the normal menstrual cycle: A. FSH promotes luteal phase development B. Progestogen is produced by granulose cells. C. Oestrogen is produced mainly by theca cells D. Prostaglandins stimulate myometrial contraction. E. Progesterone peaks in the follicular phase



19.



Which of the following may cause pelvic pain? A. Retroverted mobile uterus B. Ovarian hyperstimulation C. Primary syphilis D. Policystic ovary syndrome E. Endometrial polyp.



20.



Regarding the polycystic ovary syndrome:



21.



22.



23.



A.



SHBG levels are increased



B.



Androgens are decreased



C.



LH secretion is decreased



D.



Is not related with infertility



E.



Menstrual irregularity is a common feature



Which one is notinclude in the side effects of treatment for endometriosis except : A.



Acne



B.



Hot flushes



C.



Decrease in breast size



D.



Endometrial hyperplasia



E.



Weight gain



Which of the following statements is not true? A.



Removal of both ovaries cures premenstrual syndrome



B.



Visceral pain from the uterus, fallopian tubes and ovaries is transmitted via the autonomic nervous system (T10-L1)



C.



Cone biopsy of the cervix can cause secondary dysmenorrhoea



D.



Marker of Ca-125 can be used as diagnostic tool of endometriosis.



E.



Endometriosis related with cancer.



Which of the following is not true regarding the IUD? A.



Serum copper levels are unchanged



B.



The levonorgestrel-containing IUD is associated with a small increase in the haemoglobin level



C.



Pregnancy with IUD insitu should be terminated.



24.



25.



26.



27.



D.



The copper-containing IUD is associated with a small decreased in the haemoglobin level



E.



The plastic frame of the levonorgestrel-containing IUD contains barium so that it can be seen on X-ray



Which of the following is not caused by endometriosis? A.



Haemoptysis



B.



Infertility



C.



Renal failure



D.



Cancer



E.



Dispareunia.



Which of the following statements regarding the POP (progestogen only pill) are true? A.



It works mainly by inhibiting ovulation



B.



The dose of progestogen is higher than that in the combined pill



C.



It does not appear in the breast milk



D.



It does not influence motility of sperm



E.



It is suitable for both diabetic and hypertensive women



Regarding the combined oral contraceptive (COC) pill: A.



The incidence of iron deficiency anaemia is slightly increase



B.



The COC is protective against both ovarian and endometrial cancer



C.



It does not carries an increased risk of venous thromboembolism



D.



The incidence of the benign breast disease is increased by 30%



E.



It is not suitable for young women for postponing pregnancy.



Which of the following is not true? A.



Injectable progestogens inhibit lactation



B. Post-coital IUD insertion is effective if fitted within 7 days of unprotected intercourse C.



The injectable progestogen depo-provera inhibits ovulation



D.



Return of fertility may be delayed following removal of a contraceptive implant



E.



Implant contraceptive method is suitable for post partum women.



28.



Regarding contraception and the menopause:



A. women



The low-dose COC can be used up to the menopause in healthy non-smoking



B.



Measurement of FSH is an accurate indicator of fertility



C.



HRT can be used as contraception



D.



Sterilization affects the timing of menopause



E. For pre-menopausal women, serum FSH concentration is not considered for taking COC.



29.



30.



31.



32.



Regarding the post-coital test: A.



It must be done on day 21 of the cycle



B.



It can detect anovulation



C.



It can detect tubal problems



D.



It can detect azoospermia



E.



It can detect acrosome reaction of sperm.



Which of the following provide evidence of ovulation? A.



Day 14 progesterone level



B.



Endometrial biopsy



C.



Normal menstrual cycle.



D.



Estrogen / progestogen ratio



E.



LH / FSH ratio



Which of the following is not used to correct anovulation? A.



Bromocriptine



B.



Weight loss



C.



HCG injection



D.



Low-dose oestrogen



E.



Clomiphen citrate



Which of the following may not cause male infertility?



33.



34.



A.



Gonorrhoea infection



B.



Cystic fibrosis



C.



Prolactinoma



D.



Hydrocele



E.



Varicocele



Which of the following are associated with early pregnancy loss? A.



Sexual intercourse



B.



Multiple partner



C.



Wart virus infection



D.



Pulmonal tuberculosis



E.



Intra-uterine pregnancy with an intra-uterine device in situ



Regarding osteoporosis, which one is not true:



A. Hip fracture causes more deaths than cancer of the cervix, uterus and ovaries combined B.



Bone densitometry can be used to screen individual at risk



C.



May be associated with hypogonadism



D.



Fracture is more common on stratum compactum instead of trabecular.



E. process.



35.



36.



Osteocyt : osteoblasts that are trapped in cortical bone during the remodeling



Which the following are associated with an increased risk of osteoporosis? A.



Malabsorption syndrome



B.



Steroid therapy



C.



Hyperparathyroidism



D.



Late menopause



E.



Premature ovarian failure (POF)



Case-1



A 32-year-old nulliparous woman has had oligomenorrhea since menarche. During the past 5 years she has experienced slow but progressive increase in hair on her face, back and forearms in the inter-mammary space, and on the back of her hands. Her voice has slowly deepened and temporal balding and clitoromegaly have developed. Which of the following is the most likely diagnosis?



37.



A.



Polycystic ovary syndrome



B.



Cushing’s syndrome



C.



Stromal hyperthecosis



D.



Ovarian tumor



E.



Asherman’s syndrome



Case-2. A couple have a 2-year history of infertility. The male partner is healthy with no past serious illnesses, and has one child from his previous marriage. The female partner has a regular 29-day menstrual cycle and has a normal body mass index. Her only previous pregnancy was an ectopione. Which of the following investigations is most likely to reveal the cause of the infertility? A.



Semen analysis



B.



Pelvic ultrasound examination



C.



Hormonal profile



D.



Hysteroscopy



E.



Laparoscopy



Case-3 (questions 38 – 40) A 47-year-old patient presents wondering if her problems with mood, swings, insomnia, and vaginal dryness represent menopause. She had a hysterectomy 10 years ago for abnormal uterine bleeding, but the uterus were not removed. Since she cannot afford hormonal testing, a maturation index is done on her pap smear.



38.



Ideally, cytologic cells for evaluation of hormonal status should be obtained from the A.



Ectocervix



B.



Endocervix



C.



Labia minora



D.



Lateral vaginal wall



E.



Posterior vaginal fornix.



39.



40.



Ninety percent of the cells found on her pap smear have thick, rounded cytoplasm and plump, round, vesicular nuclei with an intact chromatin patter. The maturation index (MI) would most likely be A.



90 / 0 / 10.



B.



90 / 10 / 0



C.



10 / 0 / 90



D.



10 / 90 / 0.



E.



0 / 90 / 10



Based on this result, one would anticipate that if hormonal levels were obtained they would show A.



Elevated estrogen level



B.



Elevated progesterone levels.



C.



Low estrogen levels



D.



Normal estrogen levels



E.



Normal progesterone levels



Case-4 A patient presented to the emergency department with an infected incomplete abortion. During the dilatation and curettage (D&C) excessive bleeding developed that required vigorous curetting to control. She returns to the physician 6 months later complicating that she has not had a menstrual cycle. She has all the symptoms of getting ready to start a period but never sees any bleeding.



41.



42.



This history implies that what layer of endometrium is damaged ? A.



Basal zone



B.



Compact zone



C.



Functional zone



D.



Spongy zone



E.



None of above



Infertility can be caused by infection of  



43.



44.



45.



46.



A.



Staphylococcus



B.



Streptococcus



C.



Chlamydia



D.



Candidaalbicans



E.



Trichomonas



Müllerian agenesis A.



is the second most common cause of primary amenorrhea



B.



is associated with an abnormal karyotype



C.



no associated with spinal skeletal anomalies



D.



requires laparoscopic diagnosis



E.



no ovulation



Which of the following substances stimulates GnRH secretion? A.



Beta-endorphin



B.



Dopamine



C.



Dynorphin



D.



Norepinephrin



E.



Serotonin



Leiomyomata A.



Are suppress to grow when exposed to progestins



B.



Are infrequently associated with endometrial hyperplasia



C.



Can be reduced in size when treated with RU486



D.



Are a frequent cause of infertility



E.



Treatment option always hysterectomy



The three principal androgens in decreasing order of potency are A.



Androstenedione, testosterone,dihydrostestosterone



B.



Dihydrostestosterone, androstenedione, testosterone



C.



Dihydrostestosterone, testosterone, androstenedione



D.



Testosterone, androstenedione, dihydrostestosterone



E.



Testosterone,dihydrostestosterone, androstenedione



47.



48.



49.



50.



Which of the following is not used to correct anovulation? A.



Bromocriptine



B.



Weight loss



C.



HCG injection



D.



Low-dose oestrogen



E.



Clomiphen citrate



Which of the following may not cause male infertility? A.



Gonorrhoea infection



B.



Cystic fibrosis



C.



Prolactinoma



D.



Hydrocele



E.



Varicocele



Which of the following are associated with early pregnancy loss? A.



Sexual intercourse



B.



Multiple partner



C.



Wart virus infection



D.



Pulmonal tuberculosis



E.



Intra-uterine pregnancy with an intra-uterine device in situ



Regarding osteoporosis, which one is not true: A. Hip fracture causes more deaths than cancer of the cervix, uterus and ovaries combined B.



Bone densitometry can be used to screen individual at risk



C.



May be associated with hypogonadism



D.



Fracture is more common on stratum compactum instead of trabecular.



E. Osteocyt : osteoblasts that are trapped in cortical bone during the remodeling process.



1. You see five postmenopausal patients in the clinic. Each patient hasone of the conditions listed, and each patient wishes to begin hormonereplacement therapy today. Which patient would you start on therapy atthe time of this visit? a. Mild essential hypertension b. Liver disease with abnormal liver function tests c. Malignant melanoma d. Undiagnosed genital tract bleeding e. Treated stage III endometrial cancer



2. A 9-year-old girl presents for evaluation of regular vaginal bleeding.History reveals thelarche at age 7 and adrenarche at age 8. The most commoncause of this condition in girls is a. Idiopathic b. Gonadal tumors c. McCune-Albright syndrome d. Hypothyroidism e. Tumors of the central nervous system 3.Which of the following is a true statement regarding the psychologicalsymptoms of the climacteric? a. They are considerably less important than hormone levels b. They commonly include insomnia, irritability, frustration, and malaise c. They are related to a drop in gonadotropin levels d. They are not affected by environmental factors e. They are primarily a reaction to the cessation of menstrual flow 4. Osteoporosis is least likely in which of the following women? a. Asian b. White c. Smokers d. Sedentary e. Obese 5. Which of the following is consistent with a diagnosis of delayedpuberty? a. Breast budding in a 10-year-old girl b. Menarche delayed beyond 16 years of age c. Menarche 1 year after breast budding d. FSH values less than 20 mIU/mL e. Obese 6.An 18-year-old consults you for evaluation of disabling pain with hermenstrual periods. The pain has been present since menarche and isaccompanied by nausea and headache. History is otherwise unremarkable,and pelvic examination is normal. You diagnose primary dysmenorrhea and recommend initial treatment with which of the following? a. Ergot derivatives b. Antiprostaglandins c. Gonadotropin-releasing hormone (GnRH) analogues d. Danazol e. Codeine 7. Normal stature with minimal or absent pubertal development maybe seen in a. Testicular feminization b. Kallman syndrome



c. Pure gonadal dysgenesis d. Turner syndrome e. Intermittent athletic training 8.Medications used in the treatment of idiopathic central precociouspuberty include a. Exogenous gonadotropins b. Ethinyl estradiol c. GnRH agonists d. Clomiphene citrate e. Conjugated estrogens (e.g., Premarin) 9.Delayed puberty and sexual infantilism associated with hypergonadotropichypogonadism can be seen in patients with which of the following? a. Adrenogenital syndrome (testicular feminization) b. McCune-Albright syndrome c. Kallman syndrome d. Gonadal dysgenesis e. Mullerian agenesis 10.While evaluating a 30-year-old woman for infertility, you diagnosea bicornuate uterus. You explain that additional testing is necessarybecause of the woman’s increased risk of congenital anomalies in whichorgan system? a. Skeletal b. Hematopoietic c. Urinary d. Central nervous e. Tracheoesophageal 11.A 39-year-old woman, gravida 3, para 3, complains of severe, progressivesecondary dysmenorrhea and menorrhagia. Pelvic examinationdemonstrates a tender, diffusely enlarged uterus with no adnexal tenderness.Results of endometrial biopsy are normal. This patient most likely has a. Endometriosis b. Endometritis c. Adenomyosis d. Uterine sarcoma e. Leiomyoma 12.The most important indication for surgical repair of a double uterus,such as a septate or bicornuate uterus, is a. Habitual abortion b. Dysmenorrhea c. Menometrorrhagia d. Dyspareunia e. Premature delivery 13.In an amenorrheic patient who has had pituitary ablation for a craniopharyngioma,which of the following regimens is most likely to result inan ovulatory cycle? a. Clomiphene citrate b. Pulsatile infusion of gonadotropin-releasing hormone (GnRH) c. Continuous infusion of GnRH d. Human menopausal or recombinant gonadotropin e. Human menopausal or recombinant gonadotropin followed by human chorionicgonadotropin (hCG)



14. In the evaluation of a 26-year-old patient with 4 months of secondaryamenorrhea, you order serum prolactin and β-hCG assays. Thepregnancy test is positive, and the prolactin comes back at 100 ng/mL (normal10 mm. b. The need a sensitive pregnancy test. c. a well-timed serum progesterone determination, can help to document ovulation or anovulation. d. If a very thin endometrial stripe (less than 5 mm), suggests an attenuated or denuded endometrium. e. Hysteroscopy is the definitive methode for intrauterine pathology.



56.



Regarding endoscopy gynecology, which one of the following statements is INCORRECT?   a. A hemodynamically unstable patient with the need for control of bleeding probably should be approached by laparotomy b. A trick that provide safety in gaining access to the abdomen is by avoiding Trendelenburg position c. It’s safe to use two different energy sources in the abdomen at the same time d. If the pressure used for peritoneal insufflation is limited, laparoscopy can be performed under conscious sedation e. Atraumatic forceps are used where trauma of the tissue are to be particularly considered



57.



Gonadotropin-releasing hormone (GnRH) stimulates the release of:   a. Adrenocorticotropic hormone (ACTH) b. Growth hormone (GH)



c. Luteinizing hormone (LH) d. Opiate peptide e. Thyroid-stimulating hormone (TSH)



58.



Considering learning curve of a laparoscopic surgeon, these procedures should be considered difficult cases, EXCEPT:   a. b. c. d. e.



Ovarian cyst size 4 cm with history of laparoscopic cholecystectomy Uterine fibroid size more than 5 cm Multiple uterine fibroid Adenomiosis for hysterectomy Deep infiltrating endometriosis



59.



To preserve the pneumoperitoneum when placing secondary trocars, it is important to:   a. Ensure the surgeon is in the proper position b. Close the stopcock on the secondary port before insertion c. Ensure the stopcock on the secondary port is open before insertion d. Ensure the blade on the trocar is sharp e. Close the insufflation stopcock on the primary port



60.



The process of ovarian ageing imply for a testing prior to Assisted Reproductive Technology   a. Female age is a poor predictor for ovarian reserve. b. Ovarian reserve can be considered normal when use of exogenous gonadotropins result in the development of 1-2 follicles. c. The endocrine marker anti-Mullerianhormon (AMH) which is produced by the granulosa cells sorrounding the antral follicles, provides a novel direct marker of quantity. d. Basal FSH levels will become decreased with advancing age. Basal FSH provides the most indirect marker. e. The antral follicle count (AFC) assessed by laparoscopy provides direct visual assessment of the cohort.



61.



Which of the following statements regarding GnRH stimulated LH secretion is accurate?   a. It is enhanced by gonadotrope exposure to progesterone b. It is increased by gonadotrope exposure to testosterone c. It is enhanced by gonadotrope exposure to estrogen d. It is enhanced by gonadotrope exposure to continuous GnRH e. It is associated with steady LH release



62.



Low density lipoprotein (LDL) cholesterol serves as the principal substrate for steroidogenesis. Which of the following statements regarding circulating LDL is correct?   a. LDL is formed after addition of triglycieride to very low-density lipoprotein (VLDL) b. LDL levels are negatively correlated with cardiovascular disease c. LDL is the major carrier of cholesterol in the plasma



d. LDLD enters the cells by passive diffusion e. LDL facilitates the transport of polar lipids in the blood plasma



63.



Which of the following events that can lead to antispermantibodies ? a. b. c. d. e.



64.



Which of the following events that can lead to azoospermia with the result of Fructose test (-) in semen?   a. b. c. d. e.



65.



Testicular feminization Kallman syndrome Pure gonadal dysgenesis Turner syndrome Intermittent athletic training



Medications used in the treatment of idiopathic central precociouspubertyinclude   a. b. c. d. e.



67.



Congenital A VasDeferens (CAVD) Orchitis Klinefelter syndrome Chronic Prostatitis Varicocele



Normal stature with minimal or absent pubertal development maybe seen in   a. b. c. d. e.



66.



pulmonary tuberculosis nephrotic syndrome lupus eritematous hepatitis parotitis 



Exogenous gonadotropins Ethinyl estradiol GnRH agonists Clomiphene citrate Conjugated estrogens



While evaluating a 30-year-old woman for infertility, you diagnoseabicornuate uterus. You explain that additional testing is necessary because of the woman’s increased risk of congenital anomalies in which organ system?   a. b. c. d. e.



Skeletal Hematopoietic Urinary Central nervous Tracheoesophageal



68.



A 39-year-old woman, gravida 3, para 3, complains of severe, progressivesecondary dysmenorrhea and menorrhagia. Pelvic examinationdemonstrates a tender, diffusely enlarged uterus with no adnexal tenderness.Results of endometrial biopsy are normal. This patient most likely has   a. b. c. d. e.



69.



A 45-year-old woman who had two normal pregnancies 15 and 18years ago presents with the complaint of amenorrhea for 7 months. She expresses the desire to become pregnant again. After exclusion of pregnancy,which of the following tests is next indicated in the evaluation of this patient’s amenorrhea?   a. b. c. d. e.



70.



Antiestrogens Gonadotropins Phenothiazines Prostaglandins GnRH analogues



A 9-year-old girl has breast and pubic hair development. Evaluation demonstrates a pubertal response to a gonadotropin-releasing hormone(GnRH) stimulation test and a prominent increase in luteinizing hormone (LH) pulses during sleep. These findings are characteristic of patients with   a. b. c. d. e.



72.



Hysterosalpingogram Endometrial biopsy Thyroid function tests Testosterone and DHAS levels FSH & Estradiol levels



A 23-year-old woman presents for evaluation of a 7-month history of amenorrhea. Examination discloses bilateral galactorrhea and normal breast and pelvic examinations. Pregnancy test is negative. Which of the following classes of medication is a possible cause of her condition?   a. b. c. d. e.



71.



Endometriosis Endometritis Adenomyosis Uterine sarcoma Leiomyoma



Theca cell tumors Iatrogenic sexual precocity Premature thelarche Granulosa cell tumors Constitutional precocious puberty



The following is true regarding early relationship of hypothalamus-pituitary-ovarian axis  



a. The first initiation is prior to puberty b. Initiated with FSH increase. c. At puberty, central hypersensitivity to estrogen is decline. d. Soon will lead to regular ovulation. e. Resulting in the first folliculogenesis.



73.



Woman with the gonadal dysgenesis :   a. Have normal serum FSH level between age 8-16 years b. Have normal serum FSH level between age 2-6 years c. Have lowest serum FSH level consistent with normal menopaused values a teenagers d. Demonstrate augmented gonadotropin secretion during wake up e. Have normal serum LH level between age 8-16 years



74.



A couple with male infertility characterized by a semen analysis with a sperm count of 14 milion/mL, 25% motility, and 23% normal forms presents to your clinic. The husband’s physical examination and hormone studies are normal. The appropriate initial therapy is :   a. Clomiphene citrate b. Varicocelectomy c. In vitro fertilization (IVF) d. Intrauterine insemination with washed husband’s sperm e. Fallopian sperm tube perfusim



75.



A 9-year-old girl presents for evaluation of regular vaginal bleeding.History reveals thelarche at age 7 and adrenarche at age 8. The most commoncause of this condition in girls is   a. b. c. d. e.



76.



Idiopathic Gonadal tumors McCune-Albright syndrome Hypothyroidism Tumors of the central nervous system



Which of the following pubertal events is not mediated by gonadal estrogen production and therefore would occur even in the absence of estrogen production?   a. Breast development b. Menstruation c. Pubic hair growth



d. Skeletal growth e. Vaginal cornification



77.



During an evaluation for infertility, a woman may have an endometrial biopsy to evaluate the quality of her ovulation since the development of the corpus luteum is most closely associated with the:   a. b. c. d. e.



78.



Which statement best describes estrogen positive feedback on LH release?   a. b. c. d. e.



79.



Fertilization of an ovum Follicular phase of the endometrium Proliverative phase of the endometrium Secretory phase of the endometrium Shedding phase of the endometrium (menstruation)



It is affected by the level of circulating estrogen It is enhanced by testosterone It is increased by opioid peptides It is unaffected by progesterone It is unaffected by the duration of estrogen stimulation



The simplest, yet most useful, initial test to begin her evaluation would be   a. Serum estrogen level b. Prolactin c. Thyroid index d. Serum follicle-stimulating hormone (FSH) and luteinizing hormone (LH) e. A cardiogram



80.



The histology of adenomyosisshows   a. The metaplastic change of glandular epithelium to muscle fibers in the uterus b. The same pattern and location as endometriosis c. The presence of endometrial glands and stroma deep within uterine muscle d. A premalignant change of the endometrium e. A premalignant change of the uterine muscle



81.



An increased risk of Osteoporosis is associated with:   a. Bromocriptine use b. Excess glucocorticosteroid usage c. Androgen excess in the female.



d. Early menarche e. Excessive protein intake



82.



Recognized feature of postmenopausal women is:   a. b. c. d. e.



83.



Low FSH & LH Increased bone mineral density. High FSH High level of estrogen Increase the incidence of uterine fibroid



Which of the following is a true statement regarding the psychological symptoms of the climacteric?  



a. They are considerably less important than hormone levels. b. They commonly include insomnia, irritability, frustration, and malaise c. They are related to a drop in gonadotropin levels d. They are not affected by environmental factors e. They are primarily a reaction to the cessation of menstrual flow



84.



Which of the following contraceptive methods should NOT be used by a patient with coronary heart disease?   a. b. c. d. e.



Combined oral contraceptive pills Male condom Female condom Diaphragm Spermicidal agent



Soal No. 85-87 Mrs N 28 yo, with 5 years infertility, prolonged cycle, LMP 2 months ago. She treated with MPA 10 mg twice daily for 10 days, but there was not bleeding. Sperm analysis normozoospermia. Her body weight is 90 kg, with 155 cm height, blood pressure 160/100, and pulse 80/menit. her pubic hair growth upward until umbilicus. Gynecological examination within normal limit. The result of urine pregnancy test was negative. 85.



Based on clinical data the most possible diagnose of this case is, infertility with,   a. Unovulation WHO group I and obesity. b. Unovulation WHO group II and obesity c. PCOs, obesity, and metabolic syndrome d. Non classic CAH e. Cushing’s syndrome



86.



To confirm the previous clinical diagnose these examination have to do.   a. b. c. d. e.



87.



Serum LH. Serum FSH. Serum Free Testosterone Serum DHEAS. TVS.



The first line regimen for ovulation induction is,   a. b. c. d. e.



Clomiphen citrate Aromatase inhibitor Recombinant FSH Metformine Laparoscopic Ovarian Drilling



Soal No. 88-90 Mrs P 40 yo with prolonged bleeding, consulted by GP. She has no child. There is history of chronic abnormal uterine bleeding, no history of contraception or medication. Physical examination :155 cm height, 80 kg weight, blood pressure 160/100, pulse rate 100/menit, Hb 9 g%. Gynecological examination within normal limit. TVS uterus in normal size, endometrial thickness 14 mm, PCO feature on both side of ovary. 88.



The initial management in this case is.   a. b. c. d. e.



89.



Progestin treatment. High dose COCs Estrogen high dose, followed by COCs Curettage with histopathological examination GnRHa depot



This statement is correct about hormonal treatment in this case, EXCEPT   a. Progestin enhance conversion estradiol to estrone b. Estrogen has coagulation effect c. GnRHa depot can stop bleeding faster d. The goal is estrogen-progestin withdrawal bleeding e. Against oncogenic effect of estradiol



90.



This case has high risk of,   a. b. c. d. e.



Malignancy of the ovary Malignancy of myometrium DM type I Metabolic syndrome Early menopause



Soal No.; 91-93 Mrs D 35 yo has 2 children use low dose COCs started 2 months later, consulted by GP with intermenstrual spotting. No history of missing pill, also no history of other medication. She taking COCs contained 20 mcg estradiol combined with 3 mg drospirenone, 24 pills and 4 placebo. Blood pressure 130/80, pulse rate 80/m, 160 cm height, 60 kg weight. Physical examination and gynecological examination within normal limit 91.



The most possibility causal of intermenstrual spotting in this case is,   a. Very low dose of estradiol. b. The side effect of drospirenone c. The composition of packed 24 pills and 4 placebo d. Accidental serum FSH increase e. Accidental serum LH increase.



92.



These statement is correct about this COCs, EXCEPT,   a. Drospirenone against the LH surge b. Drospirenone have anti androgenic effect c. Estrogen component enhance the endometrial stability d. Estrogen component cause increasing serum potassium e. Lower dose estrogen against the blood coagulation effect



93.



Benefit of this COCs is, EXCEPT   a. b. c. d. e.



94.



Not cause dyslipidemia Not increase retain of water Improve elevated blood pressure High effectiveness Improve heavy menstrual bleeding



The following is true regarding the situation of early cycle   a. The LH receptor is in granulosa cells b. FSH stimulates the expression of of LH receptor c. Estradiol and FSH stimulate LH receptor d. Progesteron stimulates the secretion center of gonadotropin e. Estradiol inhibits the secretion center of gonadotropin



95.



The following is true regarding oocytes reserve   a. The highest oocytes reserve is at the time of parturition b. The development of primordial follicle disappear for sometimeafter parturition. c. At menarche there are 6 to 7 million oocytes d. Pregnancy will influence the decline of oocytes reserve e. Stimulation drugs inhance the decline of oocytes reserve



96.



Which of the following is the best explaination for breast development in a patient with androgen insensitivity ?   a. b. c. d.



Gonadal production of estrogen Adrenal production of estrogen Breast tissue sensitivity to progesterone Peripheral convertion of androgens



e. Autonomous production of breast-specific estrogen



97.



Steroid hormones are classifies as:   a. b. c. d. e.



98.



Amino acids Phospholipids Lipids Glycoprotein None of the above



Leptin and reproduction   a. Leptin administration does not accelerates the onset of puberty in rodents. b. Leptin levels decrease at puberty. c. Low leptin levels are present in athletes and in patients which anorexia d. Leptin levels are greater in males than females e. Leptin levels are increase in postmenopausal women



99.



While in the in vivo environment embryo is maintained their viability by cell sorrounding in the fallopian tubes, in vitro condition of human embryos:   a. Glucose is needed in large amount of cleavage stage human embryo b. Genomic imprinting of the human embryo take place at pronuclear stage of development c. Glucose is not an important component in the culture system since human embryo are using mainly amino acid for their energy source



d. Human embryo exhibits a considerable degree of plasticity enabling it to develop under a wide variety of culture condition e. Human culture system is performed in room temperature with sterile environment



100. Low estriol level in pregnancy correlates with;   a. b. c. d. e.



Fetal macrosomia P450c17 deficiency Adrenal hypofunction Adrenal hyperfunction 5-OH-steroid dehydrogenase deficiency



1. The differences between RNA and DNA are, except : f. g. h. i. j.



RNA uses pentose sugar RNA is single stranted RNA contain uracil Sugar in RNA is ribose RNA carries codon



What happen in increase Corticotrophin Releasing Hormone (CRH) in fetal development, All statement in bellow are true; except : A. Increase fetal cortisol is positive feedback relationship B. Augment fetal ACTH secretion. C. Adjunct progesterone secretion. D. More DHEAS. E. Increase estrogen



2. Which regard of the following is related to the first sign of pubertal development of female : f. Menarche g. Menopause h. Pubarche i. Menstruation j. Telarche 3. Mechanism of action for steroid hormones : A. Rapidly transported across the cell membrane by osmose action. B. All of which require direct interaction with DNA. C. Via hormone-receptor complex to bind hormone-responsive element in DNA. D. Not regulate posttranscriptional events. E. Translation via DNA methylation.



4. This statement is not true about basic mechanisms of menstruation f. g.



Menstruation is preceeded by intense vasoconstriction of the spiral arterioles Ischaemia induced by intense vasoconstriction of 4 to 24 hours will be augmented by increased release of free oxygen radicals produced by tissue (endometrial) desquamation h. Platelet fibrin plug formation is grossly deficient in endometrium i. Ruptured spiral arterioles do not form platelet fibrin thrombi in the early phases of menstruation



j.



The rapid formation of fibrin plugs and their degradation play a part of the control of menstrual bleeding



5.The purpose of the progestational challenge test is to asses the level of endogenous : F. Testosterone G. Progesterone H. Estrogen I. Prolactin J. TSH 6. Which of the following sequences best describes estrogen action? a. cell membrane diffusion, steroidreceptor,-DNA complex formation, transcription, translation b.cell membrane receptor activation, steroidreceotor DNA complex formation, translation,transcription, transcription c. cell membrane diffusion' steroidreceptor-DNA complex formation, translation, transcription d.cell membrane diffusion, adenylatecyclase activation, cAMP production, protein phosphorylation e. cell membrane receptor activation, adenylatecyclase activation, c AMP production, protein phosphorilation..



7.Amenorhea should be evaluated in any patient without a periode who has an absence of growth or development of secondary sexual characteristic by age F. 8 G. 10 H. 12 I. 14 J. 16 8.Sperm capacitation refers to a process by which spermatozoa become capable of: a. stimulating meiosis of ovum b. dispersing the zona radiata c. penetrating the cervical mucus d. producing acrosomal enzymes e. fertilizing the ovum



9.Gonadotropin releasing hormone (GnRH) stimulates the release of: f.



Adrenocorticotropic hormone (ACTH)



g. h. i. j.



Growth hormone (GH) Luteinizing hormone (LH) Opiate peptide Thyroid-stimulating hormone (TSH)



10. This statement is not true about basic mechanisms of menstruation a. Menstruation is preceeded by intense vasoconstriction of the spiral arterioles b. Ischaemia induced by intense vasoconstriction of 4 to 24 hours will be augmented by increased release of free oxygen radicals produced by tissue (endometrial) desquamation c. Platelet fibrin plug formation is grossly deficient in endometrium d. Ruptured spiral arterioles do not form platelet fibrin thrombi in the early phases of menstruation e. The rapid formation of fibrin plugs and their degradation play a part of the control of menstrual bleeding



11. Which of the following refers to WHO consensus 1999 about normosperm : f. g. h. i. j.



consentration less than 20 million/ml volume at least 2 ml grade A at least 50% grade A + B more than 2% leucocyte less than 2/ml



12.Regulation of fetal adrenal gland in relation with labor process, in late gestation : A. The mother prevent high cortisol by converting to cortisone. B. Increasing estogen maternal. C. Increasing 11β- hydroxysteroid dehydrogenase. D. A, B and C true. E. A and C true. 13.29-year-old primigravida who received no prenatal care has marked vaginal bleeding after the onset of labor at 38 weeks gestation. Cesarean section is performed and a lacerated low-lying placenta is removed. She remains hypotensive for 6 hours and requires transfusion of 12 packed RBC units. Postpartum, she becomes unable to breast-feed the infant. She does not have a resumption of normal menstrual cycles. She becomes more sluggish and tired. Laboratory findings include hyponatremia, hyperkalemia, and hypoglycemia. Which of the following pathologic lesions is she most likely to have had following delivery? A 



Bilateral adrenal hemorrhage







 Pituitary necrosis







 Subacute thyroiditis







 Metastatic choriocarcinoma







 Insulitis



14. A patient presents with amenorrhea and galactorrhea. Her prolactin level are elevated. She is not and never has been pregnant. In addition to evaluating her for a prolactinoma, one also needs to evaluate for other causes that would increase prolactin such as elevated. a. dopamine b. gamma-aminobutyric acid c. hystamin type II receptor activation d. Thyrotropin releasing hormone e. corticotrophin releasing hormone



15. The initial step in the workup of amenorheic patient after excluding pregnancy begin with measurement of : F. TSH G. LH H. AMH I. FSH J. GnRH 16. Which of the following sequences best describes estrogen action? a. cell membrane diffusion, steroidreceptor,-DNA complex formation, transcription, translation b. cell membrane receptor activation, steroidreceotor DNA complex formation, translation, transcription, transcription c. cell membrane diffusion' steroidreceptor-DNA complex formation, translation, transcription d. cell membrane diffusion, adenylatecyclase activation, cAMP production, protein phosphorylation e. cell membrane receptor activation, adenylatecyclase activation, c AMP production, protein phosphorilation..



17. Physical examination of amenorhea : F. Body mass index G. Breast development H. The presence of pubic hair growth I. Evaluation of the genital outflow and uterus



J.



If all are correct



18. Which of the following causes of delayed puberty accompanies elevated circulating gonadotropin levels? a. Kallman's syndrome b. Hypothalamic tumors c. gonadaldysgenesis d. malnutrition e. chronic illness



19. Which of the following sequences best describes estrogen action? a. cell membrane diffusion, steroidreceptor,-DNA complex formation, transcription, translation b. cell membrane receptor activation, steroidreceotor DNA complex formation, translation, transcription, transcription c. cell membrane diffusion' steroidreceptor-DNA complex formation, translation, transcription d. cell membrane diffusion, adenylatecyclase activation, cAMP production, protein phosphorylation e. cell membrane receptor activation, adenylatecyclase activation, c AMP production, protein phosphorilation..



20.What happen in increase Corticotrophin Releasing Hormone (CRH) in fetal development, All statement in bellow are true; except : A. Increase fetal cortisol is positive feedback relationship B. Augment fetal ACTH secretion. C. Adjunct progesterone secretion. D. More DHEAS. E. Increase estrogen